Крок 1 - Стоматологія 2017 (буклет)

1 / 200
У населеному пункті, розташованому на березі Дніпра, виявлені випадки опісторхозу. З метою профілактики санстанція зобов’язана попередити жителів, що необхідно: In a settlement located on the bank of the Dnieper, cases of opisthorchosis were detected. For the purpose of prevention, the health center is obliged to warn residents that it is necessary:

Достатньо проварювати свинину It is enough to boil the pork

Обдавати овочі і фрукти окропом Pour vegetables and fruits with boiling water

Добре проварювати і прожарювати рибу It is good to boil and fry fish

Достатньо проварювати яловичину Enough to boil the beef

Кип’ ятити питну воду Boil drinking water

2 / 200
При обстеженні дівчини 18-ти років знайдені наступні ознаки: недорозвинення яєчників, широкі плечі, вузький таз, вкорочення нижніх кінцівок, 'шия сфінкса' розумовий розвиток не порушено. Встановлено діагноз: синдром Шерешевського-Тернера. Яке хромосомне порушення у хворої? During the examination of an 18-year-old girl, the following signs were found: underdevelopment of the ovaries, broad shoulders, narrow pelvis, shortening of the lower limbs, 'neck of the sphinx', mental development is not impaired. The diagnosis was established: Shereshevsky-Turner syndrome. What chromosomal disorder does the patient have?

Моносомія Х Monosomy X

Трисомія 13 Trisomy 13

Нульсомія Х Nulsomia X

Трисомія Х Trisomy X

Трисомія 18 Trisomy 18

3 / 200
При вивченні родоводу сім’ї, в якій спостерігається гіпертрихоз (надмірне оволосіння) вушних раковин, виявлена ознака трапляється в усіх поколіннях тільки у чоловіків і успадковується від батька до сина. Визначте тип успадкування гіпертрихозу: When studying the pedigree of a family in which hypertrichosis (excessive hairiness) of the auricles is observed, the identified symptom occurs in all generations only in men and is inherited from father to son Determine the type of inheritance of hypertrichosis:

Аутосомно-рецесивний Autosomal recessive

Зчеплений з Х-хромосомою рецесивний X-linked recessive

Зчеплений з Y-хромосомою Linked to Y-chromosome

Зчеплений з Х-хромосомою домінантний X-linked dominant

Аутосомно-домінантний Autosomal dominant

4 / 200
На клітину подіяли речовиною, яка спричинила порушення цілісності мембран лізосом. Що відбудеться з клітиною внаслідок цього? The cell was exposed to a substance that disrupted the integrity of the lysosomal membranes. What will happen to the cell as a result?

Спеціалізація Specialization

Автоліз Autolysis

Трансформація Transformation

Дегенерація Degeneration

Диференціація Differentiation

5 / 200
У хворого затримка сечі в сечовому міхурі. Яка статева залоза пошкоджена? The patient has retention of urine in the bladder. Which gonad is damaged?

Яєчко Testicle

Цибулино-сечівникова залоза Onion-urethral gland

Передміхурова залоза Prostate

Сім ’яний міхурець Seven Bubbles

Придаток яєчка Appendix

6 / 200
До відділення хірургічної стоматології надійшла новонароджена дівчинка, яка при смоктанні починала поперхува- тись. При обстеженні виявлена розщілина твердого піднебіння, яка була наслідком незрощення середнього лобового відростка з верхньощелепним відростком I-ої зябрової дуги. Розщілина знаходилась у піднебінні між: A newborn girl was admitted to the department of surgical dentistry, who began to drool when sucking. During the examination, a cleft of the hard palate was revealed, which was the result of non-union of the middle frontal process with the maxillary process 1st gill arch. The cleft was in the palate between:

Lamina horizontalis os palatinum dextrum et sinistrum Lamina horizontalis os palatinum dextrum et sinistrum

Processus palatinus maxillae et lamina horizontalis os palatinum Processus palatinus maxillae et lamina horizontalis os palatinum

В ділянці canalis incisivus In the area of canalis incisivus

Os incisivum et processus palatinus maxillae Os incisivum et processus palatinus maxillae

Processus palatinus maxillae dextrae et sinistrae Processus palatinus maxillae dextrae et sinistrae

7 / 200
У хворого фронтит. В анамнезі - запалення верхньощелепної пазухи. Через який відділ носової порожнини могла потрапити інфекція в лобову пазуху? The patient has frontitis. In the anamnesis - inflammation of the maxillary sinus. Through which part of the nasal cavity could the infection enter the frontal sinus?

Нижній носовий хід Inferior nasal passage

Присінок носової порожнини Nasal cavity

Верхній носовий хід Upper nasal passage

Середній носовий хід Average nasal passage

Решітчасто-клиноподібна кишеня Lattice-wedge pocket

8 / 200
До стоматолога звернувся хворий зі скаргами на асиметрію лиця. Лікар побачив, що на лівій половині обличчя брова стоїть нижче, на лобі немає складок, віко вужче, очне яблуко виступає вперед. Функція якої пари черепно-мозкових нервів уражена? A patient came to the dentist with complaints of facial asymmetry. The doctor saw that on the left side of the face the eyebrow is lower, there are no folds on the forehead, the eyelid is narrower, the eyeball protrudes forward. The function of which pair of cranial nerves is affected?

I I

VI VI

V V

VII VII

IV IV

9 / 200
Після травми м’яких тканин в області задньої поверхні медіального виростка плеча у постраждалого виникло відчуття поколювання шкіри медіальної поверхні передпліччя. Який з наведених нервів знаходиться в зоні ушкодження? After a soft tissue injury in the region of the back surface of the medial condyle of the shoulder, the victim developed a tingling sensation in the skin of the medial surface of the forearm. Which of the following nerves is located in the area of damage?

N. musculocutaneus N. musculocutaneus

N. dorsalis scapularis N. dorsalis scapularis

N. radialis N. radialis

N. ulnaris N. ulnaris

N. subscapularis N. subscapularis

10 / 200
На гістологічному зрізі однієї з ендокринних залоз видно округлі структури різних розмірів, стінка яких утворена одним шаром епітеліальних клітин на базальній мембрані, всередині ці структури містять гомогенну неклітинну масу. Яка це залоза? A histological section of one of the endocrine glands shows rounded structures of various sizes, the wall of which is formed by a single layer of epithelial cells on the basement membrane, inside these structures contain a homogeneous non-cellular mass. What is it a gland?

Надниркова, кіркова речовина Adrenal, cortical substance

Щитоподібна Thyroid

Задня частка гіпофізу Posterior lobe of the pituitary gland

Прищитоподібна Pythroid

Передня частка гіпофізу Anterior pituitary lobe

11 / 200
При обстеженні окуліст з’ясував, що пацієнт не розрізняє синій та зелений кольори при нормальному сприйнятті іншої кольорової гами. З порушенням функції яких структур сітківки це пов’язано? During the examination, the ophthalmologist found out that the patient does not distinguish between blue and green colors with normal perception of another color gamut. This is due to a violation of the function of which structures of the retina?

Біполярні нейрони Bipolar neurons

Горизонтальні нейрони Horizontal neurons

Паличкові нейрони Rod neurons

Колбочкові нейрони Cone neurons

Амакринні нейрони Amacrine neurons

12 / 200
Під час тренування у спортсмена була травмована нижня кінцівка. Лікар травматолог встановив діагноз: розрив сухожилка. До якого типу сполучної тканини належить тканина, що утворює сухожилок? During training, the athlete injured his lower limb. The traumatologist diagnosed a tendon rupture. To what type of connective tissue does the tendon-forming tissue belong?

Пухка волокниста Fluffy Fibrous

Щільна неоформлена волокнистої Dense unframed fibrous

Хрящова Khryaschova

Ретикулярна Reticular

Щільна оформлена волокниста Dense styled fibrous

13 / 200
При гістохімічному дослідженні лейкоцитів мазку крові визначаються клітини, у цитоплазмі яких знаходяться гранули, що містять гістамін і гепарин. Які це клітини? During the histochemical examination of leukocytes of a blood smear, cells are identified in the cytoplasm of which there are granules containing histamine and heparin. What are these cells?

Моноцити Monocytes

Еозинофіли Eosinophils

Еритроцити Erythrocytes

Базофіли Basophils

Нейтрофіли Neutrophils

14 / 200
При електронномікроскопічному дослідженні гіалінового хряща виявляються клітини з добре розвиненою гранулярною ендоплазматичною сіткою, комплексом Гольджі. Яку функцію виконують ці клітини? Electron microscopic examination of hyaline cartilage reveals cells with a well-developed granular endoplasmic reticulum, Golgi complex. What function do these cells perform?

Утворення міжклітинної речовини Formation of intercellular substance

Депонування глікогену Glycogen storage

Трофіка хрящової тканини Cartilage tissue trophic

Руйнування міжклітинної речовини хряща Destruction of cartilage intercellular substance

Депонування жиру Fat Deposit

15 / 200
Після антигенної стимуляції на гістологічному зрізі лімфовузла експериментальної тварини у мозкових тяжах знайдено велику кількість клітин такої морфології: інтенсивно базофільна цитоплазма, ексцентрично розміщене ядро з хроматином, що розташований у вигляді 'спиць колеса' та світлою ділянкою ци-топлазми біля нього. Які це клітини? After antigenic stimulation, a large number of cells with the following morphology were found on a histological section of a lymph node of an experimental animal in the brain cords: intensively basophilic cytoplasm, an eccentrically located nucleus with chromatin located in the form 'spoke of the wheel' and a bright area of cytoplasm near it. What kind of cells are these?

Плазмоцити Plasmocytes

Макрофаги Macrophages

Адипоцити Adipocytes

Фібробласти Fibroblasts

Тканинні базофіли (опасисті клітини) Tissue basophils

16 / 200
В препараті діагностується тканина, в якій клітини розміщуються поодинці та ізогрупами, а в міжклітинній речовині не видно волокнистих структур. Яка тканина присутня в препараті? A tissue is diagnosed in the preparation, in which cells are located singly and in isogroups, and fibrous structures are not visible in the intercellular substance. What kind of tissue is present in the preparation?

Гіалінова хрящова Hyaline cartilage

Гладенька м’язова Smooth muscular

Епітеліальна Epithelial

Кісткова Bone

Волокниста хрящова Fibrocartilage

17 / 200
Сполучна тканина побудована з паралельно розташованих колагенових волокон, розмежованих фібробластами. Цей тип сполучної тканини називається: Connective tissue is made of parallel collagen fibers separated by fibroblasts. This type of connective tissue is called:

Пухка Puff

Щільна неоформлена Dense unformatted

Слизова Mucous

Щільна оформлена Dense Formatted

Ретикулярна Reticular

18 / 200
Недорозвиненість яких відділів лицьового черепу в ембріональний період приводить до появи такої вади розвитку, як 'вовча паща'? Underdevelopment of which parts of the facial skull in the embryonic period leads to the appearance of such a malformation as 'wolf mouth'?

Нижньощелепні відростки Mandibular processes

Нижньощелепні і піднебінні відростки Mandibular and palatine processes

Піднебінні відростки Palates

Лобні і верхньощелепні відростки Frontal and maxillary processes

Лобні відростки Frontal processes

19 / 200
У навколопульпарному дентині декальцінованого зуба дорослої людини визначаються ділянки з невпорядкова- ним розташуванням дентинних трубочок і колагенових фібрил. Назвіть даний вид дентину: In the peripulpal dentin of a decalcified adult tooth, there are areas with a disordered arrangement of dentin tubules and collagen fibrils. Name this type of dentin:

Склерозований Sclerosis

Мертві шляхи Dead paths

Первинний Primary

Третинний (іррегулярний) Tertiary (irregular)

Вторинний (регулярний) Secondary (regular)

20 / 200
Імплантація зародка у слизову оболонку матки складається з двох фаз - адгезії та інвазії. Перша фаза супроводжується: The implantation of the embryo in the mucous membrane of the uterus consists of two phases - adhesion and invasion. The first phase is accompanied by:

Пригніченням секреції маткових залоз Suppression of secretion of uterine glands

Руйнуванням сполучної тканини ендо- МЄТРІЮ Destroying the connective tissue of the endometrium

Прикріпленням бластоцисти до поверхні ендометрію Attachment of the blastocyst to the surface of the endometrium

Руйнуванням ЄПІТЄЛІОЦИТІВ слизової оболонки (ендометрію) матки Destruction of EPITHELIOCYTES of the mucous membrane (endometrium) of the uterus

Активізацією секреції маткових залоз Activation of secretion of uterine glands

21 / 200
У хворого з пересадженим серцем при фізичному навантаженні збільшився хвилинний об’єм крові. Який механізм регуляції забезпечує ці зміни? The minute volume of blood increased during physical exertion in a patient with a transplanted heart. What regulation mechanism ensures these changes?

Парасимпатичні умовні рефлекси Parasympathetic conditioned reflexes

Катехоламіни Catecholamines

Парасимпатичні безумовні рефлекси Parasympathetic unconditioned reflexes

Симпатичні умовні рефлекси Sympathetic conditioned reflexes

Симпатичні безумовні рефлекси Sympathetic unconditioned reflexes

22 / 200
Пацієнт звернувся до стоматолога зі скаргами на металевий присмак у роті і печіння язика після протезування. Які дослідження необхідно провести з метою визначення причини? The patient went to the dentist with complaints of a metallic taste in the mouth and burning of the tongue after prosthetics. What research should be done to determine the cause?

Рентгенографія X-ray

Гальванометрія Galvanometry

Електроміографія Electromyography

Оклюзіографія Occlusiography

Мастикаціографія Mastication

23 / 200
При обробці перекисом водню слизової оболонки ротової порожнини хворого, кров пофарбувалась у коричневий колір замість піноутворення. При зниженні концентрації якого з перелічених ферментів це можливо? When treating the patient's oral mucosa with hydrogen peroxide, the blood turned brown instead of foaming. When the concentration of which of the listed enzymes is reduced, this is possible?

Псевдохолінестераза Pseudocholinesterase

Ацетилтрансфераза Acetyltransferase

Метгемоглобінредуктаза Methemoglobin reductase

Каталаза Catalase

Ілюкозо-6-фосфатдегідрогеназа Ilucose-6-phosphate dehydrogenase

24 / 200
При операції на щитоподібній залозі з приводу захворювання на Базедову хворобу помилково були видалені паращи- топодібні залози. Виникли судоми, тетанія. Обмін якого біоелемента було порушено? During an operation on the thyroid gland due to Basedov's disease, the parathyroid glands were mistakenly removed. Convulsions, tetany occurred. The exchange of which bioelement was disturbed?

Залізо Iron

Калій Potassium

Магній Magnesium

Кальцій Calcium

Натрій Sodium

25 / 200
Мати помітила темну сечу у її 5-річної дитини. Жовчних пігментів у сечі не виявлено. Встановлено діагноз: алкаптону- рія. Дефіцит якого ферменту має місце? A mother noticed dark urine in her 5-year-old child. Bile pigments were not found in the urine. The diagnosis was established: alkaptonuria. Which enzyme is deficient?

Фенілаланінгідроксилаза Phenylalanine hydroxylase

Тирозиназа Tyrosinase

Оксидаза оксифенілпірувату Oxyphenylpyruvate oxidase

Декарбоксилаза фенілпірувату Phenylpyruvate decarboxylase

Оксидаза гомогентизинової кислоти Homogentisic acid oxidase

26 / 200
Ціаністий калій є отрутою, смерть організму наступає миттєво. Назвіть, на які ферменти в мітохондріях діє цианістий калій: Potassium cyanide is a poison, the death of an organism occurs instantly. Name which enzymes in mitochondria potassium cyanide acts on:

Флавінові ферменти Flavin enzymes

НАД+ - залежні дегідрогенази NAD+ - dependent dehydrogenases

Цитохром Р-450 Cytochrome P-450

Цитохромоксидаза [аа3] Cytochrome oxidase [aa3]

Цитохром В5 Cytochrome B5

27 / 200
У хворого, що проходить курс лікувального голодування, нормальний рівень глюкози у крові підтримується, головним чином, за рахунок глюконеогенезу. З якої амінокислоти у печінці людини при цьому найбільш активно синтезується глюкоза? In a patient undergoing a course of medical fasting, the normal level of glucose in the blood is maintained mainly due to gluconeogenesis. Which amino acid is the most active in the human liver at the same time is glucose synthesized?

Ілутамінова кислота Ilutamic acid

Валін Valin

Аланін Alanine

Лізин Lysine

Лейцин Leucine

28 / 200
Хворий хворіє на цукровий діабет, що супроводжується гіперглікемією натще понад 7,2 ммоль/л. Рівень якого білка пла-зми крові дозволяє ретроспективно (за попередні 4-8 тижні до обстеження) оцінити рівень глікемії у хворого? The patient has diabetes mellitus, which is accompanied by fasting hyperglycemia over 7.2 mmol/l. The level of which blood plasma protein allows retrospectively (for the previous 4-8 weeks before the examination) to assess the patient's glycemic level?

Фібриноген Fibrinogen

С-реактивний білок C-reactive protein

Церулоплазмін Ceruloplasmin

Альбумін Albumin

Ілікозильований гемоглобін Ilicosylated hemoglobin

29 / 200
У 8-місячної дитини спостерігаються блювання та діарея після прийому фруктових соків. Навантаження фруктозою призвело до гіпоглікемії. Спадкова недостатність якого ферменту є причиною стану дитини? An 8-month-old child has vomiting and diarrhea after consuming fruit juices. The fructose load has led to hypoglycemia. An inherited deficiency of which enzyme is the cause of the child's condition?

Гексокіназа Hexokinase

Фосфофруктокіназа Phosphofructokinase

Фруктозо-1,6-дифосфатаза Fructose-1,6-diphosphatase

Фруктокіназа Fructokinase

Фруктозо-1-фосфатальдолаза Fructose-1-phosphate aldolase

30 / 200
При підвищенні концентрації чадного газу в повітрі може наступити отруєння. При цьому порушується транспортування гемоглобіном кисню від легень до тканин. Накопичення в крові якого похідного гемоглобіну є причиною цього? When the concentration of carbon monoxide in the air increases, poisoning can occur. At the same time, the transportation of oxygen from the lungs to the tissues by hemoglobin is disrupted. The accumulation of which derivative of hemoglobin in the blood is the cause of this?

Карбоксигемоглобін Carboxyhemoglobin

Оксигемоглобін Oxyhemoglobin

Метгемоглобін Methemoglobin

Іемохромоген Iemochromogen

Карбгемоглобін Carbhemoglobin

31 / 200
Надмірна концентрація глюкози в ротовій рідині при цукровому діабеті призводить до розвитку: Excessive concentration of glucose in the oral fluid in diabetes leads to the development of:

Флюорозу Fluorosis

Гіпоплазії емалі Enamel hypoplasia

Посиленої кальцифікації емалі Enhanced enamel calcification

Гіперплазії емалі Enamel hyperplasia

Множинного карієсу Multiple caries

32 / 200
У 5-ти річної дитини спостерігається недостатнє звапнування емалі, карієс зубів. Гіповітаміноз якого вітаміну зумовлює розвиток такого процесу? A 5-year-old child has insufficient enamel calcification, dental caries. Hypovitaminosis of which vitamin causes the development of such a process?

Кальциферол Calciferol

Токоферол Tocopherol

Біотин Biotin

Нікотинова кислота Nicotinic acid

Фолієва кислота Folic acid

33 / 200
З віком знижується секреторна активність привушних слинних залоз. Активність якого ферменту слини буде різко зменшуватись? With age, the secretory activity of the parotid salivary glands decreases. The activity of which salivary enzyme will decrease sharply?

Гексокіназа Hexokinase

Лізоцим Lysozyme

Фосфатаза Phosphatase

Амілаза Amylase

Мальтаза Maltase

34 / 200
В експерименті кролю ввели нефроцитотоксичну сироватку морської свинки. Яке захворювання нирок моделювалося в цьому досліді? In an experiment, a rabbit was injected with nephrocytotoxic guinea pig serum. What kidney disease was simulated in this experiment?

Хронічний пієлонефрит Chronic pyelonephritis

Гострий дифузний гломерулонефрит Acute diffuse glomerulonephritis

Хронічна ниркова недостатність Chronic renal failure

Гострий пієлонефрит Acute pyelonephritis

Нефротичний синдром Nephrotic syndrome

35 / 200
При профілактичному огляді дітей в закарпатському селищі у багатьох знайдено множинний карієс. З недостатністю якого мінералу в їжі можна пов’язати розвиток карієсу? During a preventive examination of children in a Transcarpathian village, multiple caries was found in many of them. The development of caries can be associated with the deficiency of which mineral in food?

Кобальт Cobalt

Йод Iodine

Залізо Iron

Фтор Fluorine

Молібден Molybdenum

36 / 200
У людини, яка тривалий час голодувала, розвинулись набряки. Який основний механізм виникнення цих набряків? A person who has been starving for a long time developed edema. What is the main mechanism of the occurrence of this edema?

Зменшення об’єму циркулюючої крові Decreasing the volume of circulating blood

Збільшення гідростатичного тиску венозної крові Increase in hydrostatic pressure of venous blood

Зменшення гідростатичного тиску міжклітинної речовини Reduction of intercellular substance hydrostatic pressure

Зменшення онкотичного тиску плазми крові Reduction of oncotic pressure of blood plasma

Збільшення онкотичного тиску міжклітинної речовини Increase in oncotic pressure of the intercellular substance

37 / 200
У хворого діагностовано хронічний гломерулонефрит. Внаслідок значних склеротичних змін маса функціонуючих нефронів зменшилася до 10%. Яке з перерахованих нижче порушень лежить в основі розвитку в хворого уремічного синдрому? The patient was diagnosed with chronic glomerulonephritis. As a result of significant sclerotic changes, the mass of functioning nephrons decreased to 10%. Which of the following disorders is the basis for the development of uremic syndrome in the patient?

Азотемія Azotemia

Ниркова остеодистрофія Renal osteodystrophy

Порушення осмотичного гомеостазу Disruption of osmotic homeostasis

Артеріальна гіпертензія Hypertension

Порушення водного гомеостазу Disruption of water homeostasis

38 / 200
Хворий 38-ми років скаржиться на спрагу (випиває до 8 л води на добу), поліурію, схуднення, загальну слабкість. Хворіє впродовж 6 місяців. У сечі: питома вага - 1,001, лейкоцити - 1-2 в полі зору, білок - сліди. Яка причина постійної поліурії у хворого? A 38-year-old patient complains of thirst (drinks up to 8 liters of water per day), polyuria, weight loss, general weakness. He has been ill for 6 months. In the urine: specific gravity - 1.001, leukocytes - 1-2 in the field of vision, protein - traces. What is the cause of constant polyuria in the patient?

Ураження канальців нирок Kidney tubular damage

Ураження клубочків нирок Kidney glomerular damage

Зменшення продукції АДГ Reduction of ADH production

Підвищення осмотичного тиску сечі Increased urine osmotic pressure

Підвищення онкотичного тиску сечі Increased urine oncotic pressure

39 / 200
При запальних процесах в організмі починається синтез білків 'гострої фази' Які речовини є стимуляторами їх синтезу? During inflammatory processes in the body, the synthesis of 'acute phase' proteins begins. What substances are stimulators of their synthesis?

Ангіотензини Angiotensins

Інтерлейкін-1 Interleukin-1

Інтерферони Interferons

Імуноглобуліни Immunoglobulins

Біогенні аміни Biogenic amines

40 / 200
При розтині тіла померлого у прямій і сигмоподібній кишках видно дефекти слизової оболонки неправильної форми з нерівними контурами, вони зливаються між собою, залишаючи невеликі острівці слизової оболонки, що збереглася. Про який різновид коліту можна думати? During the autopsy of the deceased, defects of the mucous membrane of an irregular shape with uneven contours are visible in the rectum and sigmoid colon, they merge with each other, leaving small islands of the preserved mucosa. What type of colitis can you think of?

Фолікулярний Follicular

Виразковий Ulcerous

Катаральний Catarrhal

Фібринозний Fibrinous

Гнійний Suppurative

41 / 200
На секції виявлено: множинні геморагічні інфаркти легень, у деяких судинах легень буруватого кольору щільні маси, які не прикріплені до стінки судин, варикозне розширення вен нижніх кінцівок, в яких наявні тромби. Про який патологічний процес йдеться? The section revealed: multiple hemorrhagic lung infarcts, in some lung vessels brownish-colored dense masses that are not attached to the vessel wall, varicose veins of the lower extremities, in which blood clots are present. What pathological process is involved?

Геморагічна бронхопневмонія Hemorrhagic bronchopneumonia

Тромбоемболія легеневої артерії Thromboembolism of the pulmonary artery

Тканинна емболія легеневої артерії Tissue embolism of the pulmonary artery

Застійний тромбоз легеневої артерії Congestive pulmonary artery thrombosis

Жирова емболія легеневої артерії Fat embolism of the pulmonary artery

42 / 200
6-річну дитину доставлено в стаціонар у стані асфіксії. У гортані виявлено сірувато-жовті плівки, що легко видаляються. Який вид запалення розвинувся? A 6-year-old child was brought to the hospital in a state of asphyxiation. Grayish-yellow films were found in the larynx that were easily removed. What kind of inflammation developed?

Фібринозне Fibrinous

Десквамативно-некротичне Desquamative-necrotic

Геморагічне Hemorrhagic

Гнійне Purulent

Катаральне Catarrhal

43 / 200
У хворого на туберкульоз в біопта- ті нирки при гістологічному дослідженні у вогнищі казеозного некрозу виявлені безпорядно розсипані дрібні зерна хроматину. Наслідком чого є виявлені зміни? Disorderly scattered small grains of chromatin were found in the center of caseous necrosis in a kidney biopsy of a patient with tuberculosis during histological examination. What are the consequences of the detected changes?

Пікноз ядер Pyknosis of kernels

Мітотична активність ядер Mitotic activity of nuclei

Апоптоз Apoptosis

Каріолізис Karyolysis

Каріорексис Karyorhexis

44 / 200
У хворого 23-х років після перенесеної ангіни розвинувся сечовий синдром (гематурія, протеїнурія, лейкоцитурія). В пункційному біоптаті нирок виявлена картина інтракапілярного проліфератив- ного гломерулонефриту, а електронно- мікроскопічно виявлені великі субепіте- ліальні депозити. Який патогенез цього захворювання? A 23-year-old patient developed a urinary syndrome (hematuria, proteinuria, leukocyturia) after a sore throat. A puncture biopsy of the kidneys revealed a picture of intracapillary proliferative glomerulonephritis, and large subepithelial deposits were detected by electron microscopy. What is the pathogenesis of this disease?

Імунокомплексний механізм Immune complex mechanism

Цитотоксична, цитолітична дія антитіл Cytotoxic, cytolytic effect of antibodies

Атопія, анафілаксія з утворенням IgE і фіксацією їх на тучних клітинах Atopy, anaphylaxis with the formation of IgE and their fixation on mast cells

Клітинно обумовлений цитоліз Cell-induced cytolysis

Гранулематоз Granulomatosis

45 / 200
В інфекційну лікарню поступив хворий з ознаками пневмонії, яка розвинулась на 6-й день захворювання грипом. Який метод найвірогідніше підтверджує грипозну етіологію пневмонії? A patient was admitted to the infectious disease hospital with signs of pneumonia, which developed on the 6th day of the flu. Which method most likely confirms the influenza etiology of the pneumonia?

Виявлення антитіл проти гемаглютинінів вірусу грипу Detection of antibodies against influenza virus hemagglutinins

Зараження курячих ембріонів Infection of chicken embryos

Дослідження парних сироваток Study of paired sera

Виявлення антигенів віруса грипу в харкотинні методом !ФА Detection of influenza virus antigens in sputum by the !FA method

Імунолюмінісцентне дослідження мазків-відбитків з носових ходів Immunoluminescent study of swabs-prints from the nasal passages

46 / 200
Від хворого на гостру кишкову інфекцію виділено вірус, який віднесено до роду ентеровірусів. Для встановлення серотипу віруса застосовують діагностичні сироватки. Ці сироватки повинні містити антитіла проти: A virus belonging to the enterovirus genus was isolated from a patient with an acute intestinal infection. Diagnostic sera are used to determine the serotype of the virus. These sera must contain antibodies against:

Білків суперкапсидної оболонки Proteins of the supercapsid shell

Неструктурних білків віруса Nonstructural virus proteins

Вірусних ферментів Viral enzymes

Білків капсиду Capsid proteins

Вірусних гемаглютинінів Viral hemagglutinins

47 / 200
В інфекційну лікарню поступив пацієнт з клінічними ознаками енцефаліту. В анамнезі - укус кліща. В реакції гальмування гемаглютинації виявлено антитіла проти збудника кліщового енцефаліту в розведенні 1:20, що НЕ Є діагностичним. Вкажіть наступні дії лікаря після одержання вказаного результату: A patient with clinical signs of encephalitis was admitted to the infectious disease hospital. There is a history of a tick bite. Antibodies against the causative agent of tick-borne encephalitis in a dilution of 1:20 were detected in the hemagglutination inhibition reaction NOT diagnostic.Indicate the following actions of the doctor after receiving the specified result:

Використати чутливішу реакцію Use more sensitive response

Повторити дослідження із сироваткою, взятою через 10 днів Repeat the study with serum taken after 10 days

Повторити дослідження з іншим діа- гностикумом Repeat the study with another diagnostic

Відхилити діагноз кліщового енцефаліту Reject diagnosis of tick-borne encephalitis

Дослідити цю ж сироватку повторно Retest the same serum

48 / 200
В анотації до препарату вказано, що він містить антигени збудника черевного тифу, адсорбовані на стабілізованих еритроцитах барана. З якою метою використовують цей препарат? In the annotation to the drug, it is indicated that it contains antigens of the causative agent of typhoid, adsorbed on stabilized erythrocytes of a ram. For what purpose is this drug used?

Для виявлення антитіл в реакції гальмування гемаглютинації To detect antibodies in the hemagglutination inhibition reaction

Для серологічної ідентифікації збудника черевного тифу For serological identification of the causative agent of typhoid

Для виявлення антитіл в реакції Відаля To detect antibodies in the Vidal reaction

Для виявлення антитіл в реакції зв’язування комплементу To detect antibodies in the complement binding reaction

Для виявлення антитіл в реакції непрямої гемаглютинації To detect antibodies in the indirect hemagglutination reaction

49 / 200
Хворому з афтами слизової оболонки призначили препарат, діючим агентом котрого є галоген, а також поверхнево активна речовина, що має дезінфікуючу, дезодоруючу дію. Застосовується для де-зінфекції неметалевого інструментарію, рук, предметів догляду за інфекційними хворими. Як антисептик використовується для лікування інфікованих ран, слизової оболонки ротової порожнини, патологічних зубоясеневих кишень, дезінфекції кореневих каналів. Визначте препарат: A patient with aphthae of the mucous membrane was prescribed a drug, the active agent of which is a halogen, as well as a surface-active substance that has a disinfecting, deodorizing effect. It is used to disinfect non-metallic instruments, hands, items for the care of infectious patients. As an antiseptic, it is used for the treatment of infected wounds, the mucous membrane of the oral cavity, pathological gum pockets, disinfection of root canals. Define the drug:

Кислота борна Boric acid

Хлоргексидину біглюконат Chlorhexidine bigluconate

Калію перманганат Potassium permanganate

Перекис водню Hydrogen peroxide

Діамантовий зелений Diamond Green

50 / 200
Для ремiнералiзуючої терапії початкового карієсу ЗУ6ІВ була призначена СІЛЬ лужного металу. Визначте препарат: Alkali metal SALT was prescribed for remineralization therapy of initial caries of ZU6IV. Specify the drug:

Натрію фторид Sodium fluoride

Калію бромид Potassium bromide

Калію хлорид Potassium chloride

Натрію бромид Sodium Bromide

Натрію хлорид Sodium Chloride

51 / 200
Для корекції артеріального тиску при колаптоїдному стані хворому було введено мезатон. Який механізм дії даного препарату? Mezaton was administered to the patient to correct blood pressure in a colaptoid condition. What is the mechanism of action of this drug?

Блокує а-адренорецептори Blocks α-adrenoceptors

Стимулює а- ^-адренорецептори Stimulates a-^-adrenoceptors

Блокує ^-адренорецептори Blocks ^-adrenoceptors

Стимулює ^-адренорецептори Stimulates ^-adrenoceptors

Стимулює а-адренорецептори Stimulates α-adrenoceptors

52 / 200
Після закапування в око крапель у хворого на глаукому розвинувся міоз і короткозорість. Внутрішньоочний тиск знизився. Яка група препаратів здатна викликати такий ефект? After instilling drops into the eye, a glaucoma patient developed miosis and myopia. The intraocular pressure decreased. What group of drugs can cause such an effect?

М-холіноміметики M-cholinomimetics

Іангліоблокатори Anglioblockers

М-холіноблокатори M-cholinergic blockers

Н-холіноміметики H-cholinomimetics

а-адреноміметики α-adrenomimetics

53 / 200
До стоматолога звернулася хвора зі скаргами на біль, відчуття печіння у ясні від гарячого, кислого, соленого, солодкого, кровоточивість ясен під час прийому їжі і чищення зубів. Діагноз: гострий катаральний гінгівіт. Оберіть препарат з в’яжучим механізмом дії: A patient came to the dentist with complaints of pain, a burning sensation in the gums from hot, sour, salty, sweet, bleeding gums while eating and brushing teeth. Diagnosis : acute catarrhal gingivitis. Choose a drug with an astringent mechanism of action:

Натрій гідрокарбонат Sodium bicarbonate

Відвар кори дуба Decoction of oak bark

Хлоргексидин Chlorhexidine

Іаласкорбін Ialascorbin

Цитраль Citral

54 / 200
В умовах запалення знижується сила місцевоанестезуючої дії новокаїну. Якою є причина порушення гідролізу солі новокаїну і вивільнення активного анестетика-основи в осередку запалення? In conditions of inflammation, the strength of the local anesthetic effect of novocaine decreases. What is the reason for the violation of the hydrolysis of the novocaine salt and the release of the active anesthetic-base in the center of inflammation?

Пригнічення карбангідрази Carbonhydrase inhibition

Локальний тканинний ацидоз Local tissue acidosis

Пригнічення окислювального фосфо- рилювання Inhibition of oxidative phosphorylation

Активація сукцинатдегідрогенази Activation of succinate dehydrogenase

Локальний тканинний алкалоз Local tissue alkalosis

55 / 200
Хворому на миготливу аритмію, в анамнезі у котрого бронхіальна астма, треба призначити протиаритмічний засіб. Який препарат з цієї групи ПРОТИПОКАЗАНИЙ хворому? An antiarrhythmic drug should be prescribed to a patient with atrial fibrillation who has a history of bronchial asthma. Which drug from this group is CONTRAINDICATED for the patient?

Новокаїнамід Novocaineamide

Верапаміл Verapamil

Ніфедипін Nifedipine

Аймалін Aimalin

Анаприлін Anaprilin

56 / 200
Дитина 5-ти років поступила в ЛОР- відділення клінічної лікарні з діагнозом: гнійне запалення середнього вуха. Захворювання розпочалось з запалення носоглотки. Через який канал (каналець) скроневої кістки інфекція потрапила в барабанну порожнину? A 5-year-old child was admitted to the ENT department of a clinical hospital with a diagnosis of purulent inflammation of the middle ear. The disease began with inflammation of the nasopharynx. Through which canal (tubule) of the temporal bones, did the infection get into the tympanic cavity?

Сонно-барабанні Sleep-drumming

Барабанний Drum

Барабанної струни Drum String

Сонний Sleepy

М’язовотрубний Musulotubular

57 / 200
При проведенні дуоденального зондування зонд не проходить зі шлунка в дванадцятипалу кишку. В якому відділі шлунка є перешкода (пухлина)? During duodenal probing, the probe does not pass from the stomach into the duodenum. In which part of the stomach is there an obstruction (tumor)?

Воротар Goalkeeper

Мала кривизна Small curvature

Дно Bottom

Кардіальний відділ Cardiac Department

Тіло Body

58 / 200
У хворого на шкірі живота, грудної клітки з’явився висип у вигляді розеол і петехій, дрібноточковий кон’юнктиваль- ний висип. Смерть настала при явищах ураження мозку. При мікроскопічному дослідженні секційного матеріалу у ЦНС (довгастий мозок, міст), у шкірі і нирках, міокарді виявлений деструктивно- проліферативний ендотромбоваскуліт. Про яке захворювання слід думати? The patient developed a rash in the form of roseolae and petechiae on the skin of the abdomen and chest, small-point conjunctival rash. Death occurred due to brain damage. Upon microscopic examination of the section material in the central nervous system (oblong medulla, pons), in the skin and kidneys, and in the myocardium, destructive-proliferative endothrombovasculitis was detected. What disease should we think about?

Сепсис Sepsis

Системний червоний вовчак Systemic lupus erythematosus

Висипний тиф Typhoid

Бруцельоз Brucellosis

Вузликовий періартеріїт Nodular periarteritis

59 / 200
Хворий тривалий час лікувався з приводу пневмонії нез’ясованої етіології, стійкої до стандартної терапії. З анамнезу встановлено, що він тривалий час знаходився у службовому відрядженні у США, де отримав травму, лікувався у шпиталі, після одужання повернувся на батьківщину. Оцінюючи анамнез, клінічну картину захворювання, лікар запідозрив у хворого СНІД. Результати якого методу лабораторної діагностики дозво- ляють підтвердити попередньо встановлений діагноз у даного пацієнта? The patient was treated for a long time for pneumonia of unknown etiology, resistant to standard therapy. From the anamnesis, it was established that he was on a business trip in the USA for a long time, where received an injury, was treated in the hospital, after recovery he returned to his homeland. Evaluating the anamnesis, the clinical picture of the disease, the doctor suspected the patient of AIDS. The results of which method of laboratory diagnosis allow to confirm the previously established diagnosis in this patient?

Реакція Вiдаля Vidal's reaction

РГГА - реакція гальмування гемаглютинації RGHA - hemagglutination inhibition reaction

Імуноферментний аналiз Enzyme immunoassay

Реакщя зв’язування комплементу Complement binding reaction

Електронна мікроскопія Electron Microscopy

60 / 200
У дитячому садку через кілька годин після вживання сирної маси майже у всіх дітей раптово з’явилися симптоми гастроентериту. При бактеріологічному дослідженні блювотних мас та залишків сирної маси було виділено золотистий стафілокок. Як доцільно продовжити дослідження для уточнення джерела інфекції? In kindergarten, a few hours after eating cottage cheese, almost all children suddenly developed symptoms of gastroenteritis. Bacteriological examination of vomitus and remnants of cottage cheese revealed golden Staphylococcus. How appropriate should we continue the research to clarify the source of the infection?

Визначити здатність штамів до токси- ноутворення Determine the ability of strains to produce toxins

Провести дослідження обладнання харчоблоку Conduct research on the equipment of the food block

Провести фаготипування виділених штамів Perform phagotyping of selected strains

Вивчити наявність антитіл у хворих дітей To study the presence of antibodies in sick children

Поставити алергічну пробу Send allergy test

61 / 200
Хворий 67-ми років впродовж 20-ти років страждав на гіпертонічну хворобу. Помер від хронічної ниркової недостатності. Який вид мали нирки при розтині тіла? A 67-year-old patient suffered from hypertension for 20 years. He died of chronic kidney failure. What did the kidneys look like at autopsy?

Великі з множинними тонкостінними кістами Large with multiple thin-walled cysts

Великі строкаті Large Variegated

Великі білі Big Whites

Маленькі, щільні, поверхня дрібнозерниста Small, dense, fine-grained surface

Великі червоні Big Reds

62 / 200
При обстеженні у юнака 16-ти років було виявлено прискорення серцебиття під час вдиху, сповільнене - під час видиху. На ЕКГ відмічалося: вкорочення інтервалу RR під час вдиху та подовження його під час видиху. Назвіть вид аритмії: During the examination of a 16-year-old boy, an acceleration of the heartbeat during inspiration was detected, a slowdown - during expiration. The ECG showed: a shortening of the RR interval during inspiration and its lengthening during exhalation. Name the type of arrhythmia:

Синусова брадикардія Sinus bradycardia

Синусова тахікардія Sinus tachycardia

Ідіовентрикулярний ритм Idioventricular rhythm

Миготлива аритмія Atrial fibrillation

Синусова аритмія Sinus arrhythmia

63 / 200
Для полегшення вправлення вивиху в плечовому суглобі з метою розслаблення м’язів хворому був введений міорела- ксант дитилін. При цьому наступило 'а- пное' Що необхідно ввести хворому? To facilitate the reduction of a dislocation in the shoulder joint in order to relax the muscles, the patient was administered the myorelaxant ditilin. At the same time, apnea occurred. What should be administered to the patient ?

Галантаміну гідробромід Galantamine hydrobromide

Свіжа цитратна кров Fresh Citrate Blood

Ізонітрозин Isonitrosine

Бемегрид Bemegrid

Дипіроксим Dipiroxime

64 / 200
При обстеженні хворого 35-ти років проведено гістологічне дослідження пун- ктату червоного кісткового мозку і виявлено значне зменшення кількості мегакаріоцитів. До яких змін периферичної крові це призведе? During the examination of a 35-year-old patient, a histological examination of the red bone marrow punctate was performed and a significant decrease in the number of megakaryocytes was found. What changes in the peripheral blood will this lead to?

Лейкоцитоз Leukocytosis

Тромбоцитоз Thrombocytosis

Лейкопенія Leukopenia

Тромбоцитопенія Thrombocytopenia

Агранулоцитоз Agranulocytosis

65 / 200
Аналізуються діти в одній сім’ї. Один з батьків гомозиготний по домінантному гену полідактилії, а другий - здоровий (гомозиготний по рецесивному гену). В цьому випадку у дітей проявиться закон: Children in one family are analyzed. One of the parents is homozygous for the dominant polydactyly gene, and the other is healthy (homozygous for the recessive gene). In this case, the children the law will manifest itself:

Зчеплене успадкування Linked Inheritance

Розщеплення гібридів Split hybrids

Незалежного спадкування Independent inheritance

Чистоти гамет Gamete purity

Одноманітності гібридів першого покоління Uniformities of first generation hybrids

66 / 200
У ході експерименту з внутрішньовенним уведенням різних фізіологічно активних речовин зареєстрована тахікардія. Яка з цих речовин викликала таку реакцію? During an experiment with intravenous administration of various physiologically active substances, tachycardia was registered. Which of these substances caused such a reaction?

Ацетилхолін Acetylcholine

Тироксин Tyroxin

Калікреїн Kallikrein

Інсулін Insulin

Брадикінін Bradykinin

67 / 200
У жінки з резус-негативною кров’ю А (II) групи народилася дитина з АВ (IV) групою, у якої діагностували гемолітичну хворобу внаслідок резус-конфлікту. Яка група крові можлива у батька дитини? A woman with rhesus-negative blood of group A (II) had a child with group AB (IV), who was diagnosed with hemolytic disease as a result of rhesus conflict. What blood group is possible for the child's father?

II (А), резус-позитивна II (A), Rh-positive

І (0), резус-позитивна And (0), Rh-positive

III (В), резус-позитивна III (B), Rh-positive

IV (АВ), резус-негативна IV (AB), Rh-negative

III (В), резус-негативна III (B), Rh-negative

68 / 200
До реанімації лікарні потрапив хворий з набряком легень. Який з перелічених препаратів необхідно використати для проведення форсованого діурезу? A patient with pulmonary edema was admitted to the intensive care unit of the hospital. Which of the listed drugs should be used for forced diuresis?

Еуфілін Euphilin

Фуросемід Furosemide

Спіронолактон Spironolactone

Тріамтерен Triamterene

Гідрохлортіазид Hydrochlorothiazide

69 / 200
У хворого з’явилися жовтушність шкіри, склер та слизових оболонок. У плазмі крові підвищений рівень загального білірубіну, в калі - стеркобіліну, в сечі - уробіліну. Який вид жовтяниці у хворого? The patient developed yellowing of the skin, sclera, and mucous membranes. The level of total bilirubin in the blood plasma, stercobilin in the feces, and urobilin in the urine is elevated. What type jaundice in the patient?

Хвороба Жільбера Gilbert's disease

Обтураційна Obstructive

Гемолітична Hemolytic

Паренхіматозна Parenchymatous

Холестатична Cholestatic

70 / 200
У пацієнта порушена функція нирок. Для перевірки стану фільтраційної здатності нирок йому призначено визначення кліренсу: The patient has impaired kidney function. In order to check the state of the filtration capacity of the kidneys, he is assigned clearance determination:

Глутаміну Glutamine

Індолу Indole

Креатиніну Creatinine

Сечової кислоти Uric acid

Гідрокарбонату Hydrocarbonate

71 / 200
В експерименті після обробки нервово-м’язового препарата жаби курареподібною речовиною скорочення м’яза у відповідь на електричну стимуляцію нерва зникли. Яка функція клітинної мембрани м’яза порушується курареподібними препаратами? In an experiment, after treating the neuromuscular preparation of frogs with a curare-like substance, muscle contractions in response to electrical stimulation of the nerve disappeared. What function of the cell membrane of the muscle is disturbed drug-like drugs?

Зміна проникності для різних речовин Permeability change for different substances

Створення бар’єру між середовищем клітини та навколишньою міжклітинною рідиною Creating a barrier between the cell environment and the surrounding intercellular fluid

Рецепція медіаторів у нервово- м’язовому синапсі Reception of mediators in the neuromuscular synapse

Створення електричних потенціалів по обидва боки мембрани Creation of electrical potentials on both sides of the membrane

Підтримання внутрішньої структури клітини, її цитоскелету Maintenance of the internal structure of the cell, its cytoskeleton

72 / 200
Хворий поступив у клініку зі струсом мозку. На фоні неврологічних симптомів у крові збільшується концентрація амі-аку. Яку речовину слід призначити для знешкодження цієї речовини у мозковій тканини? A patient was admitted to the clinic with a concussion. Against the background of neurological symptoms, the concentration of ami-aku in the blood increases. What substance should be prescribed to neutralize this substance in the brain tissue?

Гістамін Histamine

Нікотинова кислота Nicotinic acid

Аскорбінова кислота Ascorbic acid

Глутамінова кислота Glutamic acid

Серотонін Serotonin

73 / 200
У біоптаті з правого головного бронха чоловіка 63 р., курця, виявлено пухлину, що складається з груп атипових епітеліальних клітин, які проникають поза межі базальної мембрани слизового шару, формують 'гнізда' та тяжі, в центральних частинах яких розташовані концентричні, яскраво-еозинофільні маси - 'ра- кові перлини'. Діагностуйте захворювання: In a biopsy from the right main bronchus of a 63-year-old man, a smoker, a tumor was found, consisting of groups of atypical epithelial cells that penetrate beyond the boundaries of the basal membrane of the mucous layer, form 'nests' and rods, in the central parts of which concentric, brightly eosinophilic masses are located - 'cancer pearls'. Diagnose the disease:

Аденокарцинома Adenocarcinoma

Дрібноклітинний рак Small cell cancer

Плоскоклітинний зроговілий рак Squamous keratinized cancer

Плоскоклітинна зроговіла папілома Squamous keratinized papilloma

Плоскоклітинний незроговілий рак Squamous non-keratinous carcinoma

74 / 200
У дитини 12-ти років низький зріст при непропорційній будові тіла і розумовій відсталості. Недостатня секреція якого гормону може бути причиною таких порушень? A 12-year-old child has short stature with a disproportionate body structure and mental retardation. Insufficient secretion of which hormone can be the cause of such disorders?

Кортизол Cortisol

Тироксин Tyroxin

Соматотропін Somatotropin

Ілюкагон Ilucagon

Інсулін Insulin

75 / 200
У вагітної жінки визначили групу крові. Реакція аглютинації еритроцитів відбулася зі стандартними сироватками груп 0 (І), В (III) і не відбулася зі стандартною сироваткою групи А (II). Досліджувана кров належить до такої групи: The blood group of a pregnant woman was determined. The erythrocyte agglutination reaction occurred with standard serums of groups 0 (I), B (III) and did not occur with standard serum of group A ( II). The studied blood belongs to the following group:

- -

0 (I) 0 (I)

АВ (IV) AB (IV)

В (III) In (III)

А (II) A (II)

76 / 200
Після фармакологічної блокади іонних каналів мембрани нервового волокна потенціал спокою зменшився з -90 до -80 мВ. Які канали було заблоковано? After pharmacological blockade of ion channels of the nerve fiber membrane, the resting potential decreased from -90 to -80 mV. Which channels were blocked?

Кальцієві Calcium

Хлорні Chlorni

Натрієві Sodium

Магнієві Magnesium

Калієві Potassium

77 / 200
Хворому на ревматизм призначили нестероїдний протизапальний засіб диклофенак- натрій. Через загострення супутнього захворювання диклофенак- натрій відмінили. Яке захворювання є протипоказом до призначення диклофенаку-натрію? The patient was prescribed the non-steroidal anti-inflammatory drug diclofenac sodium for rheumatism. Due to the exacerbation of the concomitant disease, diclofenac sodium was discontinued. What disease is a contraindication to the appointment of diclofenac sodium?

Виразкова хвороба шлунка Gastric ulcer disease

Гіпертонічна хвороба Hypertensive disease

Стенокардія Angina

Цукровий діабет Diabetes

Бронхіт Bronchitis

78 / 200
У хворого із цирозом печінки відмічається стійка артеріальна гіпотензія. (АТ- 90/50 мм рт.ст.). Чим обумовлено знижен- ня артеріального тиску при такій патології печінки? A patient with cirrhosis of the liver has sustained arterial hypotension. (BP - 90/50 mmHg). What is the reason for the decrease in arterial pressure in such a pathology liver?

Зниження синтезу ангіотензиногену Decreased synthesis of angiotensinogen

Активація калікреїн-кінінової системи Activation of the kallikrein-kinin system

Надмiрна інактивація вазопресину Excessive inactivation of vasopressin

Посилення рефлекторного впливу ІЗ рецепторної зони дуги аорти Strengthening of the reflex effect of the IR receptor zone of the aortic arch

Збільшення синтезу Аа-уретичного гормону Increased synthesis of Aa-uretic hormone

79 / 200
У приймальне відділення поступив хворий із блідою шкірою, температура тіла знижена, пульс слабкий, малого наповнення, артеріальний тиск знижений. Дана симптоматика характерна для го-строго отруєння морфіном. Який із перелічених препаратів слід застосувати в першу чергу? A patient with pale skin was admitted to the reception department, the body temperature is low, the pulse is weak, with little filling, blood pressure is low. These symptoms are characteristic of acute morphine poisoning. Which of the listed drugs should be used first?

Атропіну сульфат Atropine sulfate

Кодеїну сульфат Codeine sulfate

Налоксон Naloxone

Адреналіну гідрохлорид Adrenaline hydrochloride

Ізадрин Izadrin

80 / 200
У хлопчика 5-ти років з’явилися біль під час ковтання, набряк шиї, висока температура. При огляді на мигдаликах виявлені плівки сіро-білого кольору, які знімаються з великим зусиллям. Про яке захворювання йде мова? A 5-year-old boy developed pain when swallowing, neck swelling, high temperature. Upon examination, gray-white films were found on the tonsils, which were removed with great effort. What disease are we talking about?

Скарлатина Scarlatina

Кір Measles

- -

Менінгококова інфекція Meningococcal infection

Дифтерія Diphtheria

81 / 200
При обстеженні хворого з ендокринною патологією встановлено, що в плазмі крові підвищений рівень тестостерону. Які клітини в організмі чоловіка відповідальні за продукцію цього гормону? During the examination of a patient with endocrine pathology, it was established that the level of testosterone in the blood plasma is elevated. Which cells in a man's body are responsible for the production of this hormone?

Клітини сім’яних міхурців Seminal vesicle cells

Сустентоцити сім’яників Sustentocytes of testes

Сперматогенні клітини Spermatogenic cells

Іландулоцити сім’яників Ilandulocytes of testes

Клітини передміхурової залози Prostate cells

82 / 200
Дитині віком 6-ти років, у якої запідозрено активний туберкульозний процес, проведено діагностичну реакцію Манту. Який імунобіологічний препарат при цьому було введено? A 6-year-old child, who is suspected of having active tuberculosis, underwent a Mantoux diagnostic test. What immunobiological drug was administered?

Тулярін Tularine

Туберкулін Tuberculin

Вакцина АКДП AKDP vaccine

Вакцина БЦЖ BCG vaccine

Вакцина АДП ADP vaccine

83 / 200
Хворому 68-ми років в комплекс лікування атеросклерозу, ускладненого ішемічною хворобою серця, лікар включив гіполіпідемічний засіб, який знижує вміст в крові переважно тригліцеридів. Який із вказаних препаратів було призначено хворому? To a 68-year-old patient, in the complex treatment of atherosclerosis complicated by coronary heart disease, the doctor included a hypolipidemic drug that reduces the content of triglycerides in the blood. Which of the specified drugs was prescribed for the patient?

Кислота нікотинова Nicotinic acid

Преднізолон Prednisone

Інсулін Insulin

Ілібенкламід Ilibenclamide

Фенофібрат Fenofibrate

84 / 200
Хворий звернувся до лікаря зі скаргами на ригідність м’язів, скутість рухів, постійний тремор рук. На основі обстеження лікар встановив діагноз - хвороба Паркінсона. Зробіть раціональний вибір препарату: The patient turned to the doctor with complaints of muscle stiffness, stiffness of movements, constant hand tremors. Based on the examination, the doctor established a diagnosis of Parkinson's disease. Make a rational choice of the drug :

Сибазон Sibazone

Дифенін Difenin

Фенобарбітал Phenobarbital

Леводопа Levodopa

Етосуксимід Ethosuximide

85 / 200
Хворому на ревматоїдний артрит для попередження можливого негативного впливу на слизову шлунка призначили препарат із групи нестероїдних протизапальних засобів - селективний інгібітор ЦОГ-2. Вкажіть препарат: A patient with rheumatoid arthritis was prescribed a drug from the group of nonsteroidal anti-inflammatory drugs - a selective COX-2 inhibitor to prevent a possible negative effect on the gastric mucosa. Specify the drug:

Анальгін Analgin

Целекоксиб Celecoxib

Ібупрофен Ibuprofen

Бутадіон Butadione

Ацетилсаліцилова кислота Acetylsalicylic acid

86 / 200
У хворого зі скаргами на біль у шлунку встановлено зменшення його секреторної функції, що супроводжується анемією. Нестатність якої речовини обумовлює розвиток у хворого гіповітамінозу B12 та виникнення анемії? A patient with complaints of stomach pain has been diagnosed with a decrease in his secretory function, which is accompanied by anemia. The deficiency of which substance causes the patient to develop hypovitaminosis B12 and the occurrence of anemia?

Фактор Кастла Castle Factor

Піридоксин Pyridoxine

Тіамін Thiamine

Кальциферол Calciferol

Біотин Biotin

87 / 200
В експерименті на тварині здійснили перерізку блукаючих нервів з двох боків. Як при цьому зміниться характер дихання? In an experiment, the vagus nerves were cut on two sides of the animal. How will the pattern of breathing change?

Стане поверхневим і частим Will become superficial and frequent

Стане поверхневим і рідким Will become shallow and liquid

Стане глибоким і рідким It will become deep and liquid

Дихання не зміниться Breathing will not change

Стане глибоким і частим Will become deep and frequent

88 / 200
У дитини, хворої на серпоподібно- клітинну анемію спостерігається кілька патологічних ознак: анемія, збільшена селезінка, враження шкіри, серця, нирок і мозку. Як називається цей випадок мно- жинної дії одного гена? A child with sickle cell anemia has several pathological signs: anemia, an enlarged spleen, impressions of the skin, heart, kidneys, and brain. What is this case called? - negative effect of one gene?

Полімерія Polymeria

Плейотропія Pleiotropy

Епістаз Epistasis

Кодомінування Codominance

Комплементарність Complementarity

89 / 200
У дівчинки 15-ти років виявлено блідість шкірних покровів, глосит, гінгівіт. У крові: еритроцити - 3, 3 • 1012/л, гемоглобін - 70 г/л, кольоровий показник - 0,5. У мазку крові: гіпохромія, мікроцитоз, пой- кілоцитоз. Яка анемія спостерігається у хворої? A 15-year-old girl was diagnosed with pallor of the skin, glossitis, gingivitis. In the blood: erythrocytes - 3.3 • 1012/l, hemoglobin - 70 g/ l, color indicator - 0.5. In the blood smear: hypochromia, microcytosis, poikilocytosis. What anemia is observed in the patient?

Серпоподібно-клітинна Sickle cell

Гемолітична Hemolytic

Б12-фолієводефіцитна B12-folate deficient

Залізодефіцитна Iron deficiency

Таласемія Thalassemia

90 / 200
В нефрологічній клініці у юнака 19-ти років була виявлена підвищена кількість калію у вторинній сечі. Підвищення секреції якого гормону, імовірно могло викликати такі зміни? In the nephrology clinic, a 19-year-old man was found to have an increased amount of potassium in the secondary urine. An increase in the secretion of which hormone could probably cause such changes?

Тестостерон Testosterone

Глюкагон Glucagon

Адреналін Adrenaline

Окситоцин Oxytocin

Альдостерон Aldosterone

91 / 200
В онкологічне відділення поступив хворий з підозрою на пухлину легень. При обстеженні виявили локалізацію патології в нижній частці правої легені. Скільки бронхолегеневих сегментів має ця частка? A patient was admitted to the oncology department with a suspected lung tumor. During the examination, the localization of the pathology was found in the lower lobe of the right lung. How many bronchopulmonary segments does this lobe have?

4 4

2 2

3 3

6 6

5 5

92 / 200
У жінки, що знаходиться на лікуванні з приводу тиреотоксикозу, спостерігається підвищення температури тіла. Що лежить в основі цього явища? A woman being treated for thyrotoxicosis has an increase in body temperature. What is the basis of this phenomenon?

Зниження окислення жирів у печінці Decreased oxidation of fats in the liver

Порушення дезамінування амінокислот Violation of deamination of amino acids

Порушення синтезу глікогену Glycogen synthesis disorder

Зниження утилізації глюкози тканинами Decreased utilization of glucose by tissues

Роз’єднання окисного фосфорилюван- ня Uncoupling oxidative phosphorylation

93 / 200
У хворого наприкінці весни і на початку літа при екскурсії поза місто, а іноді і в місті спостерігаються риніт і кон’юнктивіт, що виникають раптово, супроводжуються рясним витікання рідкого сли- зу з носа і сльозотечею. Який тип алергічних реакцій спостерігається у даному випадку? At the end of spring and beginning of summer, during excursions outside the city, and sometimes in the city, the patient has rhinitis and conjunctivitis that occur suddenly, accompanied by abundant discharge of liquid mucus from the nose and lacrimation. What type of allergic reactions is observed in this case?

IV тип ГУТ Type IV GUT

II тип цитотоксичний Type II cytotoxic

- -

III тип імунокомплексний Type III immunocomplex

I тип анафілактичний Type I anaphylactic

94 / 200
У жінки 29 р. виявлено вузлоподібну пухлину нижньої щелепи із вираженою деформацією щелепи, рентгенологічними ознаками руйнування кістки. При гістологічному дослідженні пухлини спостерігаються множинні атипові одно- ядерні клітини овальної форми, поодинокі гігантські багатоядерні клітини, хаотично розміщуються кісткові балочки. Діагностуйте захворювання: A 29-year-old woman was diagnosed with a nodular tumor of the lower jaw with pronounced jaw deformation, X-ray signs of bone destruction. During histological examination of the tumor, multiple atypical oval-shaped mononuclear cells were observed , single giant multinucleated cells, chaotically placed bone beams. Diagnose the disease:

Внутрішньокісткова карцинома Intraosseous carcinoma

Амелобластична фіброма Ameloblastic fibroma

Злоякісна амелобластома Malignant ameloblastoma

Остеобластокластома Osteoblastoclastoma

Фолікулярна амелобластома Follicular ameloblastoma

95 / 200
При втручанні з метою лікування вивиху нижньої щелепи лікар повинен пам’ятати про м’яз, який при скороченні відтягує назовні капсулу і суглобовий диск скронево- нижньощелепного суглоба. Який це м’яз? When intervening to treat a dislocation of the lower jaw, the doctor must remember the muscle that, when contracted, pulls the capsule and articular disc of the temporomandibular joint outward. What is it a muscle?

M. pterygoideus lateralis M. pterygoideus lateralis

M. masseter M. masseter

M. temporalis M. temporalis

M. pterygoideus medialis M. pterygoideus medialis

M. mylohyoideus M. mylohyoideus

96 / 200
Одним із методів зняття гострого болю при невралгії трійчастого нерва є створення депо анестетика в місці виходу гілок трійчастого нерва. В якій ділянці потрібно ввести анестетик для знечулен- ня першої гілки трійчастого нерва? One of the methods for relieving acute pain in trigeminal neuralgia is to create a depot of anesthetic at the exit point of the branches of the trigeminal nerve. In which area should the anesthetic be injected to anesthetize the first branch trigeminal nerve?

Надочноямковий край Suprafoveal margin

Ділянка надперенісся Section of the bridge of the nose

Орбітальний край виличної кістки Orbital edge of zygomatic bone

Підочноямковий край Subfoveal edge

Ділянка лобного відростка верхньої щелепи Area of the frontal process of the upper jaw

97 / 200
З метою серологічної діагностики інфекційного захворювання лікарю необхідно здійснити реакцію непрямої (пасивної) гемаглютинації. Що потрібно використати для постановки цієї реакції крім сироватки хворого? For the purpose of serological diagnosis of an infectious disease, the doctor needs to perform an indirect (passive) hemagglutination reaction. What should be used to perform this reaction besides the patient's serum?

Гемолітична сироватка Hemolytic serum

Еритроцитарний діагностикум Erythrocyte diagnostics

Анатоксин Anatoxin

Інтерферон Interferon

Діагностична сироватка Diagnostic serum

98 / 200
ПІСЛЯ утворення плащового дентину живлення ВНУТРІШНІХ КЛІТИН емалевого органу порушується. Який біологічний механізм забезпечує відновлення функціонального стану і живлення амелобластів? AFTER the formation of mantle dentin, the nutrition of the INTERNAL CELLS of the enamel organ is disrupted. What biological mechanism ensures the restoration of the functional state and nutrition of ameloblasts?

Ретрузія Retrusion

Інвазія Invasion

Прогресія Progression

Регресія Regression

Інверсія Inversion

99 / 200
При мікроскопічному дослідженні бі- оптата з товстої кишки виявлена пухлина з призматичного епітелію, що формує атипові залозисті структури різної форми і величини. Базальна мембрана залоз зруйнована. Клітки пухлини поліморфні, ядра гіперхромні, відзначається велика кількість патологічних мітозів. Який діа-гноз найбільш імовірний? During microscopic examination of a colon biopsy, a tumor of prismatic epithelium was found, forming atypical glandular structures of various shapes and sizes. The basement membrane of the glands is destroyed. The tumor cells are polymorphic , the nuclei are hyperchromic, there is a large number of pathological mitoses. What is the most likely diagnosis?

Солідний рак Solid cancer

Аденокарцинома Adenocarcinoma

Слизовий рак Mucosal cancer

Базальноклітинний рак Basal cell carcinoma

Недиференційований рак Undifferentiated cancer

100 / 200
При остеолатеризмі зменшується міцність колагену, що зумовлена помітним зменшенням утворення поперечних зшивок у колагенових фібрилах. Причиною цього явища є зниження активності такого ферменту: With osteolaterism, the strength of collagen decreases, which is due to a noticeable decrease in the formation of cross-links in collagen fibrils. The reason for this phenomenon is a decrease in the activity of such an enzyme:

Колагеназа Collagenase

Пролілгідроксилаза Prolylhydroxylase

Лізилоксидаза Lysyl oxidase

Лізилгідроксилаза Lysylhydroxylase

Моноаміноксидаза Monoamine oxidase

101 / 200
У маленької дівчинки 5-ти років лікар видалив зуб на якому було два корені. Який це зуб? A 5-year-old little girl had a tooth that had two roots removed by the doctor. What kind of tooth is it?

Великий кутній верхньої щелепи Major maxilla

Малий кутній верхньої щелепи Small maxilla

Малий кутній нижньої щелепи Small angular mandible

Ікло верхньої щелепи Canine of the upper jaw

Великий кутній нижньої щелепи Major angle of the lower jaw

102 / 200
У видаленій матці жінки 55-ти років патологоанатом у товщі міометрію виявив щільний вузол діаметром 5 см з чіткими межами, на розрізі тканина вузла волокниста, сіро- рожевого кольору, з хаотичним розташуванням пучків волокон. Мікроскопічно пухлина складається з гладком’язевих клітин, що утворюють пучки різної товщини, що йдуть у різних напрямках, і прошарків місцями гіаліні- зованої сполучної тканини. Яка пухлина розвинулась у хворої? In the removed uterus of a 55-year-old woman, the pathologist found a dense nodule with a diameter of 5 cm in the thickness of the myometrium with clear boundaries. On cross-section, the tissue of the nodule is fibrous, gray-pink in color, with a chaotic arrangement of bundles of fibers. Microscopically, the tumor consists of smooth muscle cells that form bundles of different thicknesses, running in different directions, and layers of hyalinized connective tissue in places. What kind of tumor developed in the patient?

Міосаркома Myosarcoma

Фібросаркома Fibrosarcoma

Рабдоміома Rhabdomyoma

Фіброміома Fibroid

Фіброма Fibroma

103 / 200
Після травми хребта у пацієнта 18- ти років впродовж місяця спостерігалась відсутність самовільного сечовипускання, яке пізніше відновилося. Який відділ спинного мозку був пошкоджений? After a spinal cord injury, an 18-year-old patient had an absence of spontaneous urination for a month, which later recovered. Which part of the spinal cord was damaged?

Грудний Breast

Поперековий Lumbar

Шийний Cervical

Поперековий і крижовий Lumbar and sacral

Крижовий Kryzhovy

104 / 200
Сечокам’яна хвороба ускладнилася виходом конкременту з нирки. На якому рівні сечоводу, найімовірніше, він може зупинитися? The urolithiasis was complicated by the release of a stone from the kidney. At what level of the ureter is it most likely to stop?

На 5 см вище тазової частини 5 cm above the pelvis

На 2 см вище впадіння в сечовий міхур 2 cm above the confluence with the bladder

На межі черевної та тазової частин On the border of the abdominal and pelvic parts

У нирковій мисці In the renal pelvis

В середній черевній частині In the middle abdomen

105 / 200
В результаті надмірної рухливості плода відбулося перетиснення пуповини, але кровообіг між плодом і матір’ю не порушився. Наявність яких структур у пуповині сприяла цьому в першу чергу? As a result of excessive mobility of the fetus, the umbilical cord was compressed, but the blood circulation between the fetus and the mother was not disturbed. The presence of which structures in the umbilical cord contributed to this in the first place?

Оболонка артерій Sheath of arteries

Оболонка вени Sheath of vein

Залишок жовткового стебельця Residual yolk stalk

Залишок алантоїса Remaining allantois

Слизова тканина Mucous tissue

106 / 200
Солдати, які отримали поранення у розпал битви, можуть не відчувати болю до її завершення. Які гормони опіатної антиноціцептивної системи зменшують відчуття болю? Soldiers wounded in the middle of battle may not feel pain until after the battle is over. What hormones of the opiate antinociceptive system reduce the sensation of pain?

Вазопресин Vasopressin

Серотоніни Serotonins

Ендорфіни Endorphins

Альдостерон Aldosterone

Окситоцин Oxytocin

107 / 200
У хворого виявлено порушення секреторної функції піднижньощелепної слинної залози. Який нерв забезпечує її вегетативну іннервацію? The patient has a violation of the secretory function of the submandibular salivary gland. What nerve provides its vegetative innervation?

N. petrosus minor N. petrosus minor

Chorda tympani Chorda tympani

N. mandibularis N. mandibularis

N. auriculotemporalis N. auriculotemporalis

N. petrosus major N. petrosus major

108 / 200
Проведено розтин тіла чоловіка, який помер від сепсису. Виявлено гнійне розплавлення тканин шиї та КЛІТКОВИНИ переднього середостіння. Гістологічно спостерігається багато дрібних абсцесів стільникової будови, у центрі яких розташовані базофільні гомогенні утвори, до яких одним кінцем прикріплені короткі паличковидні структури. Який найбільш імовірний діагноз? An autopsy was performed on the body of a man who died of sepsis. A purulent fusion of the neck tissues and anterior mediastinum CELLS was detected. Histologically, many small cellular abscesses were observed, in the center of which basophilic homogeneous formations, to which short rod-like structures are attached at one end. What is the most likely diagnosis?

Краснуха Krasnukha

Лейшманіоз Leishmaniasis

Туберкульоз Tuberculosis

Актиномікоз Actinomycosis

Дифтерія Diphtheria

109 / 200
При обстеженні хворого виявлено погіршення кровопостачання переднього відділу міжшлуночкової перегородки серця. В якій артерії знижений кровотік? During the examination of the patient, deterioration of the blood supply of the anterior part of the interventricular septum of the heart was detected. In which artery is the blood flow reduced?

Задня міжшлуночкова Posterior interventricular

- -

Передня міжшлуночкова Anterior interventricular

Огинаюча Envelope

Права вінцева Coronal right

110 / 200
При огляді ротової порожнини на вестибулярній поверхні нижнього різця зліва виявлене утворення грибоподібної форми рожевого кольору до 2 см, яке широкою ніжкою фіксоване до надальвео- лярної тканини. Під час гістологічного дослідження виявлено розгалужені судини капілярного типу з судинними бруньками, ділянками крововиливів та осередки гемосидерозу. Який найбільш імовірний діагноз? When examining the oral cavity on the vestibular surface of the lower incisor on the left, a mushroom-shaped formation of pink color up to 2 cm was found, which was fixed to the supraalveolar tissue with a wide stalk. During histological examination the research revealed branched vessels of the capillary type with vascular buds, areas of hemorrhages and foci of hemosiderosis. What is the most likely diagnosis?

Фіброзний епуліс Fibrous epulis

Фіброматоз ясен Gingival fibromatosis

Кавернозна гемангіома Cavernous hemangioma

Гігантоклітинний епуліс Giant cell epulis

Ангіоматозний епуліс Angiomatous epulis

111 / 200
У пацієнта при оцінці основного обміну, визначеного методом непрямої калориметрії необхідно врахувати належний рівень обміну речовин та енергії. Найбільш точно визначити його величину можна при врахуванні: When assessing a patient's basic metabolism, determined by the method of indirect calorimetry, it is necessary to take into account the proper level of metabolism and energy. Its value can be most accurately determined by taking into account:

Зросту й дихального коефіцієнту Height and respiratory rate

Дихального коефіцієнту й поверхні тіла Respiratory coefficient and body surface area

Дихального коефіцієнту й калоричного коефіцієнту кисню Respiration coefficient and calorific oxygen coefficient

Статі, віку, зросту й маси тіла Sex, age, height and weight

Поверхні тіла та маси Surface bodies and masses

112 / 200
При мікроскопічному дослідженні фекалій від хворого з явищами профузного проносу, багаторазового блювання та наростаючої інтоксикації, було виявлено грамнегативні палички, що нагадували кому і розташовувались групами у вигляді табунців риб. Культура збудника була виділена через середовище накопичення - 1% пептонну воду, де вона утворювала ніжну плівку. Збудник якого захворювання був виділений з фекалій хворого? During microscopic examination of feces from a patient with profuse diarrhea, repeated vomiting, and increasing intoxication, Gram-negative rods resembling coma and arranged in groups in the form of shoals of fish were found. The culture of the causative agent was isolated through the accumulation medium - 1% peptone water, where it formed a delicate film. The causative agent of which disease was isolated from the patient's feces?

Кишковий єрсиніоз Intestinal yersiniosis

Шигельоз Shigelosis

Сальмонельоз Salmonellosis

Псевдотуберкульоз Pseudotuberculosis

Холера Cholera

113 / 200
У молодої жінки діагностовано позаматкову вагітність. В якому з перерахованих органів відбувається запліднення? A young woman is diagnosed with an ectopic pregnancy. In which of the listed organs does fertilization occur?

Порожнина піхви Vaginal cavity

Порожнина матки Uterine cavity

Порожнина малого тазу Pelvic cavity

Перешийок маткової труби Isthmus of fallopian tube

Ампула маткової труби Ampoule of fallopian tube

114 / 200
Хворий на бронхіальну астму тривалий час приймає преднізолон. Який механізм дії препарату? A patient with bronchial asthma has been taking prednisolone for a long time. What is the mechanism of action of the drug?

Блокада лейкотрієнових рецепторів Blockade of leukotriene receptors

Пригнічення активності дигідрофола- тредуктази Inhibition of dihydrofolate reductase activity

Гальмування активності фосфоліпази А Inhibition of phospholipase A activity

Блокада натрієвих каналів Blockade of sodium channels

Блокада гістамінових рецепторів Blockade of histamine receptors

115 / 200
У органах хворого запідозрили паразитування декількох видів паразитів. Як називається така сукупність паразитів, що населяють організм? Several types of parasites were suspected in the patient's organs. What is the name of such a collection of parasites inhabiting the body?

Біотоп Biotope

Фітоценоз Phytocenosis

Паразитоценоз Parasitocenosis

Біогеоценоз Biogeocenosis

Екосистема Ecosystem

116 / 200
У молодої жінки виявлена аномалія рефракції ока, при якій фокусування зображення можливе за сітківкою. Як називається ця аномалія? A young woman has a refraction anomaly of the eye, in which focusing of the image is possible behind the retina. What is the name of this anomaly?

Гіперметропія Hyperopia

Астигматизм Astigmatism

Міопія Myopia

Еметропія Emetropia

Пресбіопія Presbyopia

117 / 200
У жінки, що тривало приймала антибіотики з приводу кишкової інфекції, розвинулося ускладнення з боку слизової порожнини рота у вигляді запального процесу і білого нальоту, у якому при бактеріологічному дослідженні були виявлені дріжджеподібні грибки Candida albicans. Який з перерахованих препаратів показаний для лікування цього ускладнення? A woman who had been taking antibiotics for a long time due to an intestinal infection developed complications from the mucous membrane of the mouth in the form of an inflammatory process and white plaque, in which bacteriological examination showed yeast-like fungi Candida albicans were detected. Which of the listed drugs is indicated for the treatment of this complication?

Бісептол Biseptol

Фуразолідон Furazolidone

Флуконазол Fluconazole

Поліміксин Polymixin

Тетрациклін Tetracycline

118 / 200
У чоловіка 30-ти років виявлено хронічний гастрит із підвищеною кислото- утворюючою функцією шлунка. Який препарат найбільш доцільно застосовувати для профілактики виразкової хвороби? A 30-year-old man was diagnosed with chronic gastritis with increased acid-forming function of the stomach. What drug is the most appropriate to use for the prevention of peptic ulcer disease?

Атропін Atropine

Метоклопрамід Metoclopramide

Фамотидин Famotidine

Пілокарпіну гідрохлорид Pilocarpine hydrochloride

Но-шпа No-shpa

119 / 200
Серпоподібноклітинна анемія у людини супроводжується появою в крові аномального гемоглобіну, зміною форми еритроцитів, розвитком анемії. Дане захворювання є результатом: Sickle cell anemia in a person is accompanied by the appearance of abnormal hemoglobin in the blood, a change in the shape of erythrocytes, the development of anemia. This disease is the result of:

Хромосомної аберації Chromosomal aberration

Політенії Polythenia

Мітохондріальної мутації Mitochondrial mutation

Поліплоїдії Polyploids

Генної мутації Gene mutation

120 / 200
Однією з патогенетичних ланок у розвитку променевої хвороби є інтенсифікація процесів вільно радикального окиснення речовин. Які речовини є основним джерелом утворення вільних радикалів? One of the pathogenetic links in the development of radiation sickness is the intensification of the processes of free radical oxidation of substances. What substances are the main source of the formation of free radicals?

Ліпіди Lipids

Вода Water

Гормони Hormones

Білки Proteins

Вуглеводи Carbohydrates

121 / 200
В епідермісі є клітини, що виконують захисну функцію і мають моноци- тарний генез. Які це клітини? In the epidermis there are cells that perform a protective function and have a monocytic genesis. What are these cells?

Кератиноцити остистого шару Keratinocytes of the spinous layer

Клітини Лангерганса Langerhans cells

Кератиноцити зернистого шару Keratinocytes of the granular layer

Кератиноцити базального шару Keratinocytes of the basal layer

Меланоцити Melanocytes

122 / 200
Одним з найбільш небезпечних і типових побічних ефектів антиаритмічних засобів є негативна інотропна дія. Який із перелічених антаритмічних засобів НЕ ЗМЕНШУЄ силу серцевих скорочень? One of the most dangerous and typical side effects of antiarrhythmic drugs is a negative inotropic effect. Which of the listed antiarrhythmic drugs does NOT reduce the force of heart contractions?

Дигоксин Digoxin

Метопролол Metoprolol

Верапаміл Verapamil

Пропранолол Propranolol

Хінідину сульфат Quinidine sulfate

123 / 200
У хворого встановлений діагноз: заглотковий абсцес. Куди може поширитися гнійне запалення? The patient has been diagnosed with a nasopharyngeal abscess. Where can purulent inflammation spread?

Передтрахеальний простір шиї Pretracheal space of the neck

Середнє нижнє середостіння Medium inferior mediastinum

Надгруднинний міжфасціальний простір шиї Suprasternal interfascial neck space

Переднє нижнє середостіння Anterior inferior mediastinum

Заднє нижнє середостіння Posterior inferior mediastinum

124 / 200
У хворого 26-ти років з ознаками колі-ентериту, виділено чисту культуру бактерій, яка за морфологічними, куль- туральними та біохімічними властивостями віднесена до роду шигел. Яку з названих реакцій доцільно застосувати для серологічної ідентифікації збудника? In a 26-year-old patient with signs of coli-enteritis, a pure culture of bacteria was isolated, which according to morphological, cultural and biochemical properties is classified as Shigella. Which of the mentioned reactions, is it advisable to use them for serological identification of the pathogen?

Аглютинації Agglutinations

Зв’язування комплементу Complement binding

Гальмування гемаглютинації можливих збудників захворювання Inhibition of hemagglutination of possible pathogens

Непрямої гемаглютинації Indirect hemagglutination

Преципітації Precipitation

125 / 200
При гістологічному дослідженні ділянки тканини виявлені явища каріопікнозу, каріорексису, каріолізису у ядрах клітин, а також плазмоліз - у цитоплазмі клітин. Який патологічний процес має місце у даному випадку? During a histological examination of a tissue section, the phenomena of karyopyknosis, karyorrhexis, karyolysis in the nuclei of cells, as well as plasmolysis - in the cytoplasm of cells were detected. What pathological process is taking place in this case?

Некроз Necrosis

Гіаліноз Hyalinosis

Дистрофія Dystrophy

Атрофія Atrophy

Апоптоз Apoptosis

126 / 200
У судово-медичних, антропологічних та археологічних дослідженнях для визначення віку людини аналізують загальну масу та товщину тканини, яка впродовж усього життя відкладається на поверхні дентина кореня зуба. Про яку тканину йдеться? In forensic, anthropological and archaeological studies, to determine the age of a person, the total mass and thickness of the tissue that is deposited on the dentine surface of the tooth root throughout life is analyzed. About which what kind of fabric are you talking about?

Пухка сполучна Loose connector

Цемент Cement

Щільна сполучна Dense connective

Кісткова Bone

Емаль Enamel

127 / 200
У ЖІНКИ 42-х РОКІВ має МІСЦЄ цукровий дiабет із підвищеною концентрацією глюкози в крові натще (11,5 ммоль/л). Яке з перелічених порушень буде характерне для цього захворювання? A 42-YEAR-OLD WOMAN has diabetes mellitus with an increased fasting blood glucose concentration (11.5 mmol/l). Which of the listed disorders will be characteristic of of this disease?

Аміноацидурія Aminoaciduria

Респіраторний ацидоз Respiratory acidosis

Глюкозурія Glucosuria

Гіперкапнія Hypercapnia

Метаболічний алкалоз Metabolic alkalosis

128 / 200
Аналізується каріотип жіночого організму з синдромом трисомії - (47, ХХХ). При складанні ідіограми у цьому наборі буде така кількість пар гомологічних хромосом: The karyotype of a female organism with trisomy syndrome - (47, XXX) is analyzed. When compiling the idiogram, this set will contain the following number of pairs of homologous chromosomes:

23 пари 23 pairs

21 пара 21 pairs

47 пар 47 pairs

22 пари 22 pairs

24 пари 24 pairs

129 / 200
Після початку лікування туберкульозу легень, хворий звернувся до лікаря зі скаргами на появу червоних сліз та сечі. Який препарат міг викликати такі зміни? After starting treatment for pulmonary tuberculosis, the patient turned to the doctor complaining of the appearance of red tears and urine. What drug could have caused such changes?

Рифампіцин Rifampicin

Цефазолін Cefazolin

Бензилпеніциліну калієва сіль Benzylpenicillin potassium salt

Бензилпеніциліну натрієва сіль Benzylpenicillin sodium salt

Бісептол-480 Biseptol-480

130 / 200
Хворий звернувся до стоматолога із симптомами запалення слизової оболонки ротової порожнини. У мазках, отриманих з пародонтальних кишень, виявлено найпростіші з непостійною формою тіла, розміром 6-60 мкм, здатні утворювати псевдоподії. Які це найпростіші? The patient went to the dentist with symptoms of inflammation of the mucous membrane of the oral cavity. In smears obtained from periodontal pockets, protozoa with an irregular body shape, 6-60 μm in size, were found able to form pseudopodia. What are the simplest?

Lamblia intestinalis Lamblia intestinalis

Entamoeba gingivalis Entamoeba gingivalis

Entamoeba coli Entamoeba coli

Entamoeba histolytica Entamoeba histolytica

Trichomonas hominis Trichomonas hominis

131 / 200
Жінці, яка скаржиться на постійне відчуття страху, тривоги, поставлено діагноз неврозу та призначено препарат з анксіолітичною дією. Який це препарат? A woman who complains of a constant feeling of fear, anxiety, was diagnosed with neurosis and prescribed a drug with an anxiolytic effect. What is this drug?

Діазепам Diazepam

Аміназин Aminazine

Пірацетам Piracetam

Настойка женьшеню Ginseng Tincture

Кофеїн-бензоат натрію Caffeine sodium benzoate

132 / 200
До лікаря звернувся хворий із скаргами на слабкість, нудоту, недокрів’я. Хворий зазначив, що 3 місяці тому використовував в їжу свіжепосолену ікру. На який гельмінтоз могла захворіти людина? A patient came to the doctor with complaints of weakness, nausea, anemia. The patient noted that 3 months ago he used freshly salted caviar for food. What helminthiasis could he have contracted man?

Дифілоботріоз Diphyllobotriosis

Трихінельоз Trichinellosis

Теніаринхоз Taeniarhynchosis

Теніоз Taeniosis

Дикроцеоліоз Dicroceoliosis

133 / 200
16-тирічна дівчина має зріст 139 см, крилоподібну шию, нерозвинені грудні залози, первинну аменорею. Найбільш імовірно, вона має такий каріотип: A 16-year-old girl has a height of 139 cm, a wing-shaped neck, underdeveloped mammary glands, primary amenorrhea. Most likely, she has the following karyotype:

46, ХХ/46, XY 46, ХХ/46, XY

47, ХХХ 47, XXX

45, Х0 45, X0

46, ХХ 46, XX

46, XY 46, XY

134 / 200
В шліфі зуба виявляється структура, в якій розрізняють чергування світлих і темних смуг, розташованих перпендикулярно до її поверхні, а також тонкі паралельні лінії росту. Яка це структура? A structure is revealed in the tooth slide, in which alternating light and dark stripes located perpendicular to its surface, as well as thin parallel growth lines can be distinguished. What is this structure?

Пульпа Pulp

Дентин

Емаль Enamel

Безклітинний цемент Acellular cement

Клітинний цемент Cell cement

135 / 200
Порушення процесів мієлінізації нервових волокон призводить до неврологічних розладів і розумової відсталості. Такі симптоми характерні для спадкових і набутих порушень обміну: Disturbance of the processes of myelination of nerve fibers leads to neurological disorders and mental retardation. Such symptoms are characteristic of hereditary and acquired metabolic disorders:

Холестерину Cholesterol

Фосфатидної кислоти Phosphatic acid

Сфінголіпідів Sphingolipids

Вищих жирних кислот Higher fatty acids

Нейтральних жирів Neutral fats

136 / 200
У гістологічному препараті сечоводу кілька оболонок. Яким епітелієм вистелена слизова оболонка цього органу? There are several membranes in the histological specimen of the ureter. What epithelium is the mucous membrane of this organ lined with?

Багатошаровий перехідний Multilayer Transitional

Одношаровий кубічний One layer cubic

Одношаровий плаский One-layer flat

Багатошаровий плоский незроговілий Multilayer flat non-keratinized

Одношаровий призматичний з облямівкою Single-layer prismatic with a border

137 / 200
У хворого початкова стадія гінгіві- ту. Спостерігається гіперемія ясен у при- шийкових областях зубів внаслідок роз-ширення судин мікроциркуляторного русла, що приносять кров. Яка речовина тучних клітин забезпечила вказані зміни? The patient has the initial stage of gingivitis. There is hyperemia of the gums in the cervical areas of the teeth due to the expansion of the vessels of the microcirculatory channel that bring blood. What is the substance of mast cells provided the indicated changes?

Гістамін Histamine

Ендорфіни Endorphins

Субстанція Р Substance P

Ацетилхолін Acetylcholine

Адреналін Adrenaline

138 / 200
У дитини, яку годували синтетичними сумішами, з’явились ознаки недостатності вітаміну В\. В яких реакціях бере участь цей вітамін? A child who was fed with synthetic formulas showed signs of vitamin B deficiency\. In what reactions does this vitamin participate?

Декарбоксилювання амінокислот Decarboxylation of amino acids

Трансамінування амінокислот Transamination of amino acids

Гідроксилювання проліну Hydroxylation of proline

Окислювальне декарбоксилування кетокислот Oxidative decarboxylation of ketoacids

Окислювально-відновні реакції Redox reactions

139 / 200
Яка група організмів мають кільцеві та лінійні молекули ДНК, що формують хромосоми простої будови (не мають гі- стонів)? Which group of organisms have circular and linear DNA molecules that form chromosomes of a simple structure (do not have histones)?

Бактеріофаги Bacteriophages

Гриби Mushrooms

Найпростіші The easiest

Віруси Viruses

Бактерії Bacteria

140 / 200
До лікаря-стоматолога звернувся хворий зі скаргами на сухість в ротовій порожнині, як в стані спокою, так і під час прийому їжі. При обстеженні встановлено порушення секреторної діяльності під’язикової і підщелепної слинних залоз. В якому випадку може бути таке явище? The patient came to the dentist with complaints of dryness in the oral cavity, both at rest and during eating. During the examination, a violation of the secretory activity under 'Lingual and submandibular salivary glands. In what case can this phenomenon occur?

Пошкодження окорухового нерва Damage of the oculomotor nerve

Пошкодження блокового нерва Block nerve damage

Пошкодження язикоглоткового нерва Damage of the glossopharyngeal nerve

Пошкодження піднижньощелепного нерва Injury of the submandibular nerve

Пошкодження вегетативних волокон барабанної струни лицевого нерва Damage to autonomic fibers of the tympanic cord of the facial nerve

141 / 200
У потерпілого травма верхньої щелепи, вибитий перший малий кутній зуб. Який відросток верхньої щелепи пошкоджений? The victim has an upper jaw injury, the first small canine tooth is knocked out. Which process of the upper jaw is damaged?

Піднебінний Palate

Альвеолярний Alveolar

Лобовий Frontal

- -

Виличний High

142 / 200
Хвора 18-ти років впродовж декількох тижнів скаржиться на біль та крово- точивість в області 35 зуба при вживанні твердої їжі. Об’єктивно: на жувальній поверхні цього зуба - велика каріозна порожнина, яка заповнена м’язоподібною тканиною. При зондуванні виникає кровотеча і біль в області з’єднання каріозної порожнини з пульпою. Який діагноз найбільш імовірний? An 18-year-old patient complains of pain and bleeding in the area of tooth 35 when eating solid food for several weeks. Objectively: on the chewing surface of this tooth - a large carious cavity, which is filled with muscle-like tissue. During probing, bleeding and pain occur in the area where the carious cavity meets the pulp. What is the most likely diagnosis?

Хронічний фіброзний пульпіт Chronic fibrous pulpitis

Хронічний глибокий карієс Chronic deep caries

Хронічний гіпертрофічний пульпіт Chronic hypertrophic pulpitis

Гострий гнійний пульпіт Acute purulent pulpitis

Хронічний гангренозний пульпіт Chronic gangrenous pulpitis

143 / 200
Із щелепної кістки пацієнта 45-ти років оперативно видалено порожнисте утворення діаметром - 2,5 см, заповнене жовтуватою рідиною з кристалами холестерину, яке було розташоване у верхівці кореня каріозного зуба. При мікроскопічному дослідженні внутрішня поверхня порожнини вкрита багатошаровим плоским епітелієм з акантозом, без керати- нізації. Стінка порожнини фіброзна, з кристалами холестерину, інфільтрована лімфоцитами, плазмоцитами, макрофагами, що резорбують жири. Який діагноз найбільш імовірний? From the jaw bone of a 45-year-old patient, a hollow formation with a diameter of 2.5 cm, filled with a yellowish liquid with cholesterol crystals, which was located at the apex of the carious root, was surgically removed tooth. Upon microscopic examination, the inner surface of the cavity is covered with a multi-layered flat epithelium with acanthosis, without keratinization. The cavity wall is fibrous, with cholesterol crystals, infiltrated by lymphocytes, plasma cells, macrophages that resorb fats. What is the most likely diagnosis?

Фолікулярна кіста Follicular cyst

Радикулярна кіста Radicular cyst

Амелобластома Ameloblastoma

Остеобластокластома Osteoblastoclastoma

Кератокіста Keratocyst

144 / 200
Хворий звернувся до лікаря зі скаргами на періодичні висипання герпети- чних пухирців на лінії губ і на крилах носа. Такий стан спостерігається впродовж 10-ти років, кожний раз після зниження захисних сил організму. Лікар встановив діагноз: лабіальний герпес. Як називається така форма інфекції? The patient turned to the doctor with complaints of periodic eruptions of herpetic blisters on the line of the lips and on the wings of the nose. This condition has been observed for 10 years, each time after a decrease in the body's defenses. The doctor made a diagnosis: labial herpes. What is this form of infection called?

Латентна Latent

Гостра Acute

Персистенція Persistence

Затяжна Prolonged

Екзогенна Exogenous

145 / 200
У хворого флегмона у ділянці крило-піднебінної ямки з ураженням однойменного вегетативного вузла. Функція якої залози буде порушена у першу чергу? The patient has phlegmon in the area of the pterygoid fossa with damage to the vegetative node of the same name. The function of which gland will be disturbed first of all?

Під’язикова Sublingual

- -

Навколовушна Over-the-ear

Слізна Tear

Піднижньощелепна Mandibular

146 / 200
У хворого травма у ділянці передньої поверхні переднього драбинчастого м’яза. Функція якого нерва може бути порушена? The patient has an injury in the area of the anterior surface of the anterior scalene muscle. The function of which nerve may be impaired?

Додатковий Additional

Плечового сплетення Brachial plexus

ЗВОРОТНІЙ гортанний REVERSE guttural

Діафрагмальний Aperture

Блукаючий Wandering

147 / 200
При ОГЛЯДІ ротової порожнини хворого виявлено карієс коронки зуба, яка звернена до власне ротової порожнини. Яка поверхня уражена? During EXAMINATION of the patient's oral cavity, caries of the crown of the tooth, which faces the oral cavity itself, was revealed. Which surface is affected?

Facies mesialis Facies mesialis

Facies distalis Facies distalis

Facies lingualis Facies lingualis

Facies contactus Facies contactus

Facies vestibularis Facies vestibularis

148 / 200
У хворого порушено формування відчуття солодкого та солоного смаку на ВЄРХІВЦІ та по краях язика. ЯКІ сосочки язика уражені? The patient has a violation of the formation of the sensation of sweet and salty taste on the TOP and on the edges of the tongue. WHICH papillae of the tongue are affected?

Papillae vallatae Papillae vallatae

Papillae foliatae Papillae foliatae

Papillae fungiformes Papillae fungiformes

Papillae conicae Papillae conicae

Papillae filiformes Papillae filiformes

149 / 200
Хворий 46-ти років з ревматичним стенозом помер від хронічної легенево- серцевої недостатності. На розтині виявлені щільні коричневого кольору легені. Який пігмент зумовив забарвлення легенів? A 46-year-old patient with rheumatic stenosis died of chronic pulmonary and heart failure. The autopsy revealed dense brown lungs. What pigment caused the color of the lungs?

Іемозоїн Iemosoin

Меланін Melanin

Ліпофусцин Lipofuscin

Порфірин Porphyrin

Іемосидерин Iemosiderin

150 / 200
Велику частину коронки, шийки і кореня зуба складає дентин, товщина якого з віком може збільшуватися, можливе також його часткове відновлення після пошкодження. Які структури забезпечують ці процеси? A large part of the crown, neck and root of the tooth is made up of dentin, the thickness of which can increase with age, and its partial recovery after damage is also possible. What structures provide these processes?

Дентинні канальці Dentin tubules

Амелобласти Ameloblasts

Одонтобласти Odontoblasts

Перитубулярний дентин Peritubular dentine

Цементобласти Cement areas

151 / 200
Хворий скаржиться, що при згадуванні про минулі трагічні події в його житті, у нього виникають тахікардія, задишка і різкий підйом артеріального тиску. Які структури ЦНС забезпечують за-значені кардіореспіраторні реакції у даного хворого? The patient complains that when remembering past tragic events in his life, he has tachycardia, shortness of breath, and a sharp rise in blood pressure. What structures of the central nervous system provide the specified cardiorespiratory reactions in this patient?

Чотиригорбкове тіло середнього мозку Quadriform body of the midbrain

Мозочок Cerebellum

Специфічні ядра таламуса Specific nuclei of the thalamus

Латеральні ядра гіпоталамуса Lateral nuclei of the hypothalamus

Кора великих півкуль Cortex of large hemispheres

152 / 200
Хворий звернувся з відчуттям серцебиття після стресу. ЧСС- 104/хв., тривалість інтервалу P — Q - 0,12 сек., QRS - без змін. Який тип аритмії у хворого? The patient complained of palpitations after stress. Heart rate - 104/min, duration of the P — Q interval - 0.12 sec., QRS - unchanged. What type of arrhythmia in the patient?

Синусова тахікардія Sinus tachycardia

Екстрасистолія Extrasystole

Синусова аритмія Sinus arrhythmia

Миготлива аритмія Atrial fibrillation

Синусова брадикардія Sinus bradycardia

153 / 200
Провідну роль в процесі кальцифіка- ції тканин зуба відіграє білок остеокальцин, який має високу здатність зв’язувати іони кальцію, завдяки наявності в поліпе- птидному ланцюзі залишків модифікованої амінокислоти: The leading role in the process of tooth tissue calcification is played by the protein osteocalcin, which has a high ability to bind calcium ions, due to the presence of modified amino acid residues in the polypeptide chain :

Аланін Alanine

Y-аміномасляна Y-aminobutyrate

^-амінопропіонова ^-aminopropionic

Карбоксиаспарагінова Carboxyaspartic acid

7-карбоксиглутамінова 7-carboxyglutamine

154 / 200
У плазмі крові пацієнта зпідвищи- лась активність ізоферментів ЛДІ і ЛДГ2. Про патологію якого органа це свідчить? The activity of LDI and LDH2 isozymes increased in the patient's blood plasma. What organ pathology does this indicate?

Нирки Kidneys

Мозок Brain

Печінка Liver

Скелетні м’язи Skeletal muscles

Міокард Myocardium

155 / 200
В медичній практиці застосовують антикоагулянти, що посилюють дію інгі- бітора факторів коагуляції антитромбіну III. Такий ефект притаманний: In medical practice, anticoagulants are used that enhance the action of the inhibitor of antithrombin III coagulation factors. This effect is inherent in:

Колагену Collagen

Дерматан-сульфату Dermatan sulfate

Іепарину Heparin

Кератан-сульфату Keratan sulfate

Гіалуроновій кислоті Hyaluronic acid

156 / 200
У дорослої людини у стані спокою частота серцевих скорочень дорівнює 40/хв. Водієм ритму серця у людини є: The heart rate of an adult at rest is 40/min. The driver of a person's heart rate is:

Ніжки пучка Гіса His Bundle Legs

Синоатріальний вузол Sinoatrial node

Волокна Пуркін’є Purkinje fibers

Атріовентрикулярний вузол Atrioventricular node

Пучок Гіса His Bundle

157 / 200
При видаленні верхнього лівого третього моляра виникла значна кровотеча. Ураження якої артерії призвело до кровотечі? When the upper left third molar was removed, significant bleeding occurred. Damage to which artery led to the bleeding?

A. alveolaris inferior A. alveolaris inferior

A. alveolaris superioris anterioris A. alveolaris superioris anterioris

A. infraobitalis A. infraorbitalis

A. facialis A. facialis

A. alveolaris superioris posterioris A. alveolaris superioris posterioris

158 / 200
Після екстракції зуба виникла гостра коміркова кровотеча. Який засіб при внутрішньовенному введені забезпечить гемостаз? After tooth extraction, acute cellular bleeding occurred. Which intravenous drug will provide hemostasis?

Натрію хлорид Sodium Chloride

Кислота амінокапронова Aminocaproic acid

Кальцію хлорид Calcium chloride

Тромбін Thrombin

Вікасол Vikasol

159 / 200
У пацієнта після операції з застосуванням апарату штучного кровообігу з’явилася гемоглобінурія, причиною якої може бути: The patient developed hemoglobinuria after surgery with the use of an artificial blood circulation device, the cause of which may be:

Механічний гемоліз Mechanical hemolysis

Імунний гемоліз Immune hemolysis

Хімічний гемоліз Chemical hemolysis

Біологічний гемоліз Biological hemolysis

Термічний гемоліз Thermal hemolysis

160 / 200
У потерпілого спостерігається кровотеча в ділянці верхньої губи. Яка артерія пошкоджена внаслідок травми? The victim has bleeding in the area of the upper lip. Which artery is damaged as a result of the injury?

Язикова Language

Кутова Angular

Лицева Face

Поверхнева скронева Superficial temporal

Верхньощелепна Maxillary

161 / 200
Від хворого виділені нерухомі бактерії овоїдної форми з біполярним забарвленням. В організмі утворюють ніжну капсулу. На агарі утворюють колонії з мутно-білим центром, оточеним фестончатою облямівкою, що нагадують мереживо. Продукують 'мишачий токсин' Дані властивості притаманні для збудника: Immobile, ovoid-shaped bacteria with a bipolar color were isolated from the patient. They form a delicate capsule in the body. On agar, they form colonies with a cloudy white center surrounded by a scalloped border, resembling lace. They produce 'mouse toxin'. These properties are characteristic of the pathogen:

Бруцельозу Brucellosis

Сибірки Anthrax

Коклюшу Whooping cough

Чуми Plagues

Туляремії Tularemia

162 / 200
У хворого відмічається м’язова слабкість та біль в м’язах, що обумовлено порушенням утворення карнозину та ансе- рину із амінокислоти в-аланіну. В організмі людини утворення вказаної амінокислоти відбувається в процесі катаболізму: The patient has muscle weakness and pain in the muscles, which is caused by a violation of the formation of carnosine and anserine from the amino acid β-alanine. In the human body, the formation of the specified amino acid occurs in the process of catabolism:

ГМФ HMF

ІМФ IMF

УМФ UMF

АМФ AMP

ёАМФ ёАМФ

163 / 200
Хворий 33-х років, що страждає на інсулінозалежний цукровий діабет, на фоні ін’єкції інсуліну втамував спрагу великою кількістю води, що призвело до розвитку гіпоглікемічної коми. Який вид порушення водно-сольового обміну супроводжує даний стан? A 33-year-old patient suffering from insulin-dependent diabetes mellitus quenched his thirst with a large amount of water against the background of an insulin injection, which led to the development of a hypoglycemic coma. What what kind of violation of water-salt metabolism accompanies this condition?

Гіперосмолярна гіпергідратація Hyperosmolar hyperhydration

Гіперосмолярна гіпогідратація Hyperosmolar hypohydration

Гіпоосмолярна гіпогідратація Hyposmolar hypohydration

Гіпоосмолярна гіпергідратація Hyposmolar hyperhydration

Ізоосмолярная гіпергідратація Isosmolar hyperhydration

164 / 200
Хворий 84-х років страждає на паркінсонізм, одним з патогенетичних ланок якого є дефіцит медіатора в окремих структурах мозку. Якого медіатора насамперед? An 84-year-old patient suffers from parkinsonism, one of the pathogenetic links of which is a deficiency of a mediator in certain brain structures. Which mediator first of all?

Ацетилхолін Acetylcholine

Норадреналін Noradrenaline

Дофамін Dopamine

Адреналін Adrenaline

Гістамін Histamine

165 / 200
Епідеміологічні дослідження показують, що у людей з групою крові І (0), частіше, ніж у людей з рештою груп крові, виникає виразкова хвороба шлунка і 12-палої кишки. До проявів якого виду реактивності можна віднести дане явище? Epidemiological studies show that people with blood group I (0) more often than people with other blood groups develop peptic ulcer disease of the stomach and duodenum intestines. To the manifestations of what type of reactivity can this phenomenon be attributed?

Гіперергічна Hyperergic

Групова Group

Гіпергічна Hypergic

Індивідуальна Individual

Видова Species

166 / 200
При лабораторному обстеженні крові людини, яку вкусила змія, виявлено гемоліз еритроцитів, гемоглобінурію. Дія зміїної отрути зумовлена наявністю в ній ферменту: During a laboratory examination of the blood of a person bitten by a snake, hemolysis of erythrocytes, hemoglobinuria was detected. The effect of snake venom is due to the presence of an enzyme in it:

Сфінгомієліназа Sphingomyelinase

Фосфоліпаза С Phospholipase C

Фосфоліпаза А2 Phospholipase A2

Фосфоліпаза А1 Phospholipase A1

Фосфоліпаза Д Phospholipase D

167 / 200
В медичній практиці широкого застосування набули курареподібні речовини (міорелаксанти). Про яку побічну дію повинен пам’ятати лікар при їх застосуванні? In medical practice, curare-like substances (myorelaxants) have become widely used. What side effect should a doctor remember when using them?

Розслаблення дихальних м’язів Respiratory muscle relaxation

Приступи судом Seizures

Розлади мозкового кровообігу Disorders of cerebral circulation

Зупинка серця Cardiac arrest

Тромбоутворення Thrombosis

168 / 200
При ПІДЙОМІ в гори у людини збільшується частота дихання i прискорюється серцебиття. ЯКІ ЗМІНІ в КРОВІ спричиняють це? When CLIMBING mountains, a person's breathing rate increases and the heart beats faster. WHAT CHANGES IN THE BLOOD cause this?

Зниження парціального тиску О2 Decreasing O2 partial pressure

Підвищення парціального тиску С02 C02 partial pressure increase

Зниження рН Decrease in pH

Підвищення рН Raising pH

Підвищення осмотичного тиску Increased osmotic pressure

169 / 200
У досліді вивчали просторовий поріг шкірної чутливості. Він буде найбільшим у шкірі: In the experiment, the spatial threshold of skin sensitivity was studied. It will be the largest in the skin:

Гомілки Shins

Тильної поверхні кисті Back surface of the hand

Обличчя Face

Спини Backs

Плеча Shoulder

170 / 200
Чоловіку 35-ти років, хворому на атопічний дерматит, був призначений лоратадин. Визначте механізм дії цього препарату: A 35-year-old man with atopic dermatitis was prescribed loratadine. Determine the mechanism of action of this drug:

Стимулює М-холінорецептори Stimulates M-cholinergic receptors

Блокує ^-адренорецептори Blocks ^-adrenoceptors

Блокує Ні-гістамінові рецептори Blocks No-histamine receptors

Стимулює дофамінові рецептори Stimulates dopamine receptors

Блокує ГАМК-рецептори Blocks GABA receptors

171 / 200
До лікарні доставлено хворого з отруєнням бертолетовою сіллю (KCLO3). Яка форма гіпоксії розвинулася у нього? A patient with Bertolet salt poisoning (KCLO3) was brought to the hospital. What form of hypoxia developed in him?

Тканинна Fabric

Респіраторна Respiratory

Гемічна Chemical

Гіпоксична Hypoxic

Циркуляторна Circulator

172 / 200
Під час іспиту студент побачив у гістологічному препараті орган, в кірковій речовині якого розташовані скупчення лімфатичних вузликів. У мозковій речовині знаходились тяжі клітин, що відхо-дять від вузликів. Строму органа утворюють сполучна та ретикулярна тканини. Який орган досліджував студент? During the exam, the student saw in the histological preparation an organ in the cortical substance of which there are clusters of lymph nodes. In the medulla, there were strands of cells departing from the nodules. The stroma of the organ is formed by connective and reticular tissues. What organ did the student research?

Лімфатичний вузол Lymph node

Червоний кістковий мозок Red bone marrow

Селезінка Spleen

Мигдалик Amygdalik

Тимус Thymus

173 / 200
Проведене медичне обстеження населення високогірного селища. Який по казник буде характерним для цього адаптивного екологічного типу людей? A medical examination of the population of a high-altitude village has been carried out. What kind of disease will be characteristic of this adaptive ecological type of people?

Підвищення концентрації холестерину в крові Increased blood cholesterol concentration

Зростання кисневої ємності крові Increase in blood oxygen capacity

Зниження показників основного обміну Decreasing indicators of the main exchange

Зменшення рівня гемоглобіну Decreased hemoglobin

Підсилення потовиділення Increased sweating

174 / 200
Оксид азоту відіграє важливу роль у релаксації гладеньких м’язів судин і зниженні артеріального тиску, розширенні коронарних артерій. NO в організмі може утворюватись з: Nitric oxide plays an important role in relaxation of vascular smooth muscles and reduction of blood pressure, expansion of coronary arteries. NO in the body can be formed from:

Глутаміну Glutamine

Проліну Proline

Лізину Lysin

Аргініну Arginine

Метіоніну Methionine

175 / 200
Під час патологоанатомічного розтину в померлого виявлено гній у задньому середостінні. Через який простір поширився запальний процес, що був локалізований у передньому відділі шиї? During the post-mortem examination, pus was found in the posterior mediastinum of the deceased. Through what space did the inflammatory process spread, which was localized in the front part of the neck?

Міждрабинчатий Inter-ladder

Переддрабинчатий Prescaled

Передвісцеральний Previsceral

Надгруднинний Suprasternal

Позавісцеральний Extravisceral

176 / 200
Альпініст на вершині гори Ельбрус відчуває нестачу кисню, порушення дихання, серцебиття, оніміння кінцівок. Який вид гіпоксії розвинувся? A climber on the top of Mount Elbrus feels a lack of oxygen, breathing disorders, heart palpitations, numbness of limbs. What kind of hypoxia has developed?

Гемічна Chemical

Тканинна Fabric

Гіпоксична Hypoxic

Циркуляторна Circulator

Серцева Cardiac

177 / 200
У пацієнта 28-ми років, хворого на пневмонію, розвинувся набряк легень. Об’єктивно спостерігається швидке зростання глибини та частоти дихання з переважанням фази вдиху над фазою видиху, загальне збудження, розширення зіниць, тахікардія, підвищення артеріального тиску, судоми. Який стан розвинувся у хворого? A 28-year-old patient with pneumonia developed pulmonary edema. Objectively, there is a rapid increase in the depth and frequency of breathing with a predominance of the inspiratory phase over the expiratory phase, general excitement, pupil dilation, tachycardia, increased blood pressure, convulsions. What condition has the patient developed?

Другий період асфіксії Second period of asphyxiation

Третій період асфіксії The third period of asphyxiation

Гіпопное Hypopnoea

Апное Apnoea

Перший період асфіксії First period of asphyxiation

178 / 200
У приймальне відділення доставлено хворого з гострою кровотечею. У разі втрати якого об’єму циркулюючої крові може настати летальний кінець у такого хворого? A patient with acute bleeding was brought to the receiving department. In case of loss of which volume of circulating blood, such a patient may die?

75% 75%

12% 12%

3% 3%

33% 33%

50% 50%

179 / 200
У чоловіка 30-ти років множинні переломи КІСТОК КІНЦІВОК в результат автомобільної катастрофи. В стаціонар доставлений через годину після травми в важкому стані: свідомість затьмарена, шкіра бліда, вкрита липким потом, зіниці вузькі із слабкою реакцією на світло, дихання рідке, поверхневе, тони серця приглушені, пульс ниткоподібний, АТ- 60/40 мм рт.ст. Ознак внутрішньої чи зовнішньої крововтрати немає. Який стан розвинувся у хворого? A 30-year-old man has multiple fractures of the BONES OF THE LIMBS as a result of a car accident. He was brought to the hospital an hour after the injury in a serious condition: consciousness is obscured, the skin is pale, covered sticky sweat, pupils are narrow with a weak reaction to light, breathing is shallow, shallow, heart sounds are muffled, pulse is threadlike, blood pressure - 60/40 mm Hg. There are no signs of internal or external blood loss. What condition has the patient developed?

Анафілактичний шок Anaphylactic shock

Опіковий шок Burn shock

Постгеморагічний шок Posthemorrhagic shock

Травматичний шок Traumatic shock

Кардіогенний шок Cardiogenic shock

180 / 200
При мікроскопії зубного нальоту клінічно здорової дитини 10-ти років виявлені Гр+ і Гр- мікроорганізми. Який саме етап забарвлення за методом Грама дозволив віддиференціювати Гр+ бактерії від Гр-? During microscopy of dental plaque of a clinically healthy 10-year-old child, Gr+ and Gr- microorganisms were detected. Which stage of Gram staining made it possible to differentiate Gr+ bacteria from Gr -?

Обробка розчином Люголя Treatment with Lugol's solution

Обробка водним розчином фуксину Treatment with an aqueous solution of fuchsin

Обробка спиртом Alcohol treatment

Обробка генціанвіолетом Treatment with gentian violet

Обробка сірчаною кислотою Sulphuric acid treatment

181 / 200
Хворому на туберкульоз легень призначений препарат першого ряду, який спричинив розвиток невриту лицьового нерва та порушення рівноваги. Вкажіть цей лікарський засіб: A patient with pulmonary tuberculosis was prescribed a first-line drug, which caused the development of facial nerve neuritis and impaired balance. Specify this drug:

Цефазолін Cefazolin

Левоміцетин Levomycetin

Ізоніазид Isoniazid

Фуразолідон Furazolidone

Бісептол Biseptol

182 / 200
Хворому 45-ти років з діагнозом 'амебна дизентерія' призначено протипротозойний засіб з групи нітроімідазолу. Вкажіть препарат: A 45-year-old patient diagnosed with amoebic dysentery is prescribed an antiprotozoal agent from the nitroimidazole group. Specify the drug:

Цефалексин Cephalexin

Фуразолідон Furazolidone

Бійохінол Bioquinol

Поліміксин Polymixin

Метронідазол Metronidazole

183 / 200
У постраждалого - різана рана у ділянці краю нижньої щелепи попереду жувального м’яза. З рани виливається кров темно-червоного кольору. Яку судину пошкоджено? The victim has a cut wound in the area of the edge of the lower jaw in front of the masticatory muscle. Dark red blood is pouring out of the wound. What vessel is damaged?

V facialis V facialis

V lingualis V lingualis

V jugularis externa V jugularis externa

V jugularis anterior V jugularis anterior

V retromandibularis V retromandibularis

184 / 200
У постраждалого поверхневе поранення передньої ділянки шиї, спостерігається кровотеча. Кров темного кольору. Яку судину пошкоджено? The victim has a superficial wound to the front of the neck, bleeding is observed. The blood is dark in color. What vessel is damaged?

V jugularis interna V jugularis interna

A. thyroidea superior A. thyroidea superior

V jugularis externa V jugularis externa

V jugularis anterior V jugularis anterior

A. carotis externa A. carotis externa

185 / 200
Педіатр, оглядаючи порожнину рота дитини, виявив 8 зубів. Дитина розвивається нормально. Визначте вік дитини: The pediatrician, examining the child's oral cavity, found 8 teeth. The child is developing normally. Determine the child's age:

10-12 місяців 10-12 months

15-20 місяців 15-20 months

8-9 місяців 8-9 months

6-8 місяців 6-8 months

12-15 місяців 12-15 months

186 / 200
У хворого зареєстрували ЕКГ За яким її елементом лікар може оцінити процеси розповсюдження деполяризації передсердями? The patient's ECG was recorded. By what element of it can the doctor assess the processes of spreading depolarization through the atria?

Зубець S S tooth

Зубець T Teeth T

Зубець Q Prong Q

Зубець P Prong P

Зубець R R tooth

187 / 200
Глюкагон - це гормон, що підвищує рівень глюкози в крові шляхом активації: Glucagon is a hormone that increases blood glucose levels by activating:

Глікогенфосфорилази в гепатоцитах Glycogen phosphorylases in hepatocytes

Гліколізу Glycolysis

Глюкокінази Glucokinases

Глікогенсинтази Glycogen synthases

Глікогенфосфорилази в міоцитах Glycogen phosphorylases in myocytes

188 / 200
Хворий 47-ми років впродовж останніх 3-х років хворіє на туберкульоз легень, скаржиться на задишку, тяжкість в області правого боку грудної клітки; температура тіла - 37,7oC. Виявлено правосторонній ексудативний плеврит. Який тип клітин очікується у плевральному пунктаті? A 47-year-old patient has been suffering from pulmonary tuberculosis for the past 3 years, complains of shortness of breath, heaviness in the right side of the chest; body temperature is 37, 7oC. Right-sided exudative pleurisy is detected. What type of cells is expected in the pleural punctate?

Лімфоцити Lymphocytes

Атипові клітини Atypical cells

Еозинофіли Eosinophils

Нейтрофіли Neutrophils

Еритроцити Erythrocytes

189 / 200
У пацієнта при рентгенологічному обстеженні виявлено грижу міжхребцевого диска грудного відділу хребта. Який вид з’єднання МІЖ хребцями зазнав патологічних ЗМІН? During an x-ray examination, the patient was diagnosed with a herniated intervertebral disc of the thoracic spine. What type of connection between the vertebrae underwent pathological CHANGES?

Синдесмоз Syndesmosis

Синостоз Synostosis

Діартроз Diarthrosis

Геміартроз Hemiarthrosis

Синхондроз Synchondrosis

190 / 200
У наркотизованого собаки зареєстровано електрокардіограму (ЕКГ). Збільшення яких елементів ЕКГ відбулося після подразнення гілочки лівого блукаючого нерва, яка йде до серця? An anesthetized dog has an electrocardiogram (ECG). What elements of the ECG increased after irritation of the branch of the left vagus nerve that goes to the heart?

Амплітуда зубців комплексу QRS Amplitude of the waves of the QRS complex

Амплітуда всіх зубців ЕКГ Amplitude of all ECG waves

Тривалість інтервалу P-Q Duration of PQ interval

Тривалість зубця Т T wave duration

Тривалість сегменту S-T ST segment duration

191 / 200
При огляді хворого лікар- стоматолог виявив наслідки опіку сли-зової ротової порожнини агресивною речовиною. Зі слів хворого, він випадково випив концентровану оцтову кислоту. Вказані зміни відповідають поняттю: During the examination of the patient, the dentist discovered the effects of a burn of the mucous oral cavity with an aggressive substance. According to the patient, he accidentally drank concentrated acetic acid. The indicated changes correspond to the concept:

Патологічний стан Pathological condition

Хвороба Disease

Патологічна реакція Pathological reaction

Типовий патологічний процес Typical pathological process

Патологічний процес Pathological process

192 / 200
При дослідженні біологічного матеріалу пацієнта, хворого на грип, виявлено збудник, який відносять до неклітинної форми життя. У своєму складі збудник містить РНК, оточену білками оболонки. Даний збудник належить до: During the study of the biological material of a patient with influenza, a pathogen was found, which is attributed to a non-cellular life form. The pathogen contains RNA surrounded by envelope proteins. This pathogen belongs to:

Вірусів Viruses

Бактерій Bacteria

Гельмінтів Helmints

Грибів Mushrooms

Пріонів Prions

193 / 200
Хворий 53-х років страждає на множинний карієс зубів. Що з перерахованого може бути причиною даного захворювання? A 53-year-old patient suffers from multiple dental caries. Which of the following can be the cause of this disease?

Механічне подразнення зубними протезами Mechanical irritation with dentures

Гіпосалівація Hyposalivation

Стрептокок групи А Group A Streptococcus

Гіперсалівація Hypersalivation

Харчування переважно вуглеводною їжею Mainly carbohydrate diet

194 / 200
При вивченні зрізу зуба з ознаками глибокого карієсу на межі з пульпою виявлена зона замісного дентину, у якій багато одонтобластів, невелика кількість дентинних канальців, які мають широкий просвіт, розташовуються рідко і мають звивистість хода. Який загальнопатоло- гічний процес лежить в основі утворення зони замісного дентину? When examining a section of a tooth with signs of deep caries, a zone of replacement dentin was found at the pulp boundary, in which there are many odontoblasts, a small number of dentinal tubules, which have a wide lumen, are located they rarely have a tortuous gait. What general pathological process underlies the formation of a zone of replacement dentin?

Гормональна гіперплазія Hormonal hyperplasia

Захисна гіперплазія Protective hyperplasia

Робоча гіперплазія Working hyperplasia

Замісна гіперплазія Replacement hyperplasia

Патологічна регенерація Pathological regeneration

195 / 200
У постраждалого глибока різана рана у ділянці зовнішньої поверхні кута нижньої щелепи. Який м’яз при цьому буде пошкоджено? The victim has a deep cut wound in the area of the outer surface of the corner of the lower jaw. What muscle will be damaged?

M. depressor anguli oris M. depressor anguli oris

M. buccinator M. buccinator

M. orbicularis oris M. orbicularis oris

M. masseter M. masseter

M. zygomaticus M. zygomaticus

196 / 200
У хворого на слизовій оболонці піднебіння з’явилася безболісна виразка з щільними краями і сальним червоним дном. Мікроскопічно відзначається лімфо-плазмоцитарна інфільтрація, гігантські клітини типу Пирогова-Лангханса і виражений продуктивний васкуліт. Визначте вид гранульоми: The patient developed a painless ulcer on the mucous membrane of the palate with dense edges and a greasy red bottom. Microscopically, lympho-plasmacytic infiltration, giant cells of the Pirogov-Langhans type and pronounced productive vasculitis. Determine the type of granuloma:

Склеромна Scleroma

Ревматична Rheumatic

Лепрозна Leprosy

Сифілітична Syphilitic

Туберкульозна Tuberculosis

197 / 200
До лікаря звернувся пацієнт зі скаргами на біль та обмеженість зміщення нижньої щелепи праворуч. Функція якого м’язу порушена? A patient came to the doctor with complaints of pain and limited movement of the lower jaw to the right. Which muscle function is impaired?

M. temporalis dexter M. temporalis dexter

M. masseter sinister M. masseter sinister

M. ptherygoideus lateralis sinister M. ptherygoideus lateralis sinister

M. zygomaticus major M. zygomaticus major

M. ptherygoideus lateralis dexter M. ptherygoideus lateralis dexter

198 / 200
Лікар-стоматолог під час огляду пацієнта побачив 'білі зони' демінералізації зубів. Який мікроорганізм міг спричинити такі зміни емалі? The dentist saw 'white areas' of tooth demineralization during the examination of the patient. What microorganism could have caused such enamel changes?

Streptococcus mutans Streptococcus mutans

Staphylococcus aureus Staphylococcus aureus

Streptococcus pyogenes Streptococcus pyogenes

Staphylococcus epidermidis Staphylococcus epidermidis

Neisseria meningitidis Neisseria meningitidis

199 / 200
У дитини 10-ти років раптово підвищилася температура до 39oC, з’явилися нежить, кашель, світлобоязнь. Під час огляду на слизовій оболонці порожнини рота виявлені плями Філатова-Копліка. Який з діагнозів найбільш імовірний? A 10-year-old child's temperature suddenly rose to 39oC, a runny nose, cough, and photophobia appeared. During the examination, Filatov's spots were found on the mucous membrane of the oral cavity. Koplika. Which of the diagnoses is the most probable?

Скарлатина Scarlatina

Краснуха Krasnukha

Мононуклеоз Mononucleosis

Вітряна віспа Chicken Pox

Кір Measles

200 / 200
На РОЗТИНІ тіла чоловіка, що помер ВІД серцевої декомпенсації, виявлено РІЗКО збільшене серце масою 960 г, ('бичаче серце'), у порожнині перикарду 90 мл солом’яного кольору рідини. Порожнини серця різко розширені, міокард в’ялий, товщина стінки лівого шлуночка - 2,3 см. Нирки зменшені у розмірах, масою по 70 г, поверхня їх дрібнозерниста. Під час мікроскопічного дослідження внутрішніх органів виявлений гіаліноз артеріол і гіпертрофія м’язового шару більш крупних артерій. Назвіть основне захворю-вання: On the AUTOPSY of the body of a man who died FROM cardiac decompensation, a GRABBLY enlarged heart weighing 960 g ('bull heart') was found, with 90 ml of salt in the pericardial cavity' light-colored liquid. The cavities of the heart are sharply dilated, the myocardium is flaccid, the thickness of the wall of the left ventricle is 2.3 cm. The kidneys are reduced in size, weighing 70 g each, their surface is fine-grained. During a microscopic examination of the internal organs, hyalinosis of the arterioles and hypertrophy of the 'ulcer layer of larger arteries. Name the main disease:

Ревматизм Rheumatism

Карідоміопатія Cardiomyopathy

Ішемічна хвороба серця Ischemic heart disease

Атеросклероз Atherosclerosis

Гіпертонічна хвороба Hypertensive disease